Medical Surgical Nursing Exams Board

You might also like

Download as pdf or txt
Download as pdf or txt
You are on page 1of 36

C.

Bradycardia and Hypertension

D. Fever and Hypertension


Medical Surgical Nursing Exam 1
SITUATION: Mr. D. Rojas, An obese 35 year old MS Professor of
SITUATION : Arthur, A registered nurse, witnessed an old woman OLFU Lagro is admitted due to pain in his weight bearing joint. The
hit by a motorcycle while crossing a train railway. The old woman diagnosis was Osteoarthritis.
fell at the railway. Arthur rushed at the scene.
6. As a nurse, you instructed Mr. Rojas how to use a cane. Mr. Rojas
1. As a registered nurse, Arthur knew that the first thing that he will do has a weakness on his right leg due to self immobilization and
at the scene is guarding. You plan to teach Mr. Rojas to hold the cane

A. Stay with the person, Encourage her to remain still and A. On his left hand, because his right side is weak.
Immobilize the leg while While waiting for the
ambulance.
B. On his left hand, because of reciprocal motion.
B. Leave the person for a few moments to call for help.
C. On his right hand, to support the right leg.
C. Reduce the fracture manually.
D. On his right hand, because only his right leg is weak.
D. Move the person to a safer place.
7. You also told Mr. Rojas to hold the cane
2. Arthur suspects a hip fracture when he noticed that the old woman’s
A. 1 Inches in front of the foot.
leg is
B. 3 Inches at the lateral side of the foot.
A. A. Lengthened, Abducted and Internally Rotated.
C. 6 Inches at the lateral side of the foot.
B. Shortened, Abducted and Externally Rotated.
D. 12 Inches at the lateral side of the foot.
C. Shortened, Adducted and Internally Rotated.
8. Mr. Rojas was discharged and 6 months later, he came back to the
D. Shortened, Adducted and Externally Rotated.
emergency room of the hospital because he suffered a mild stroke. The
right side of the brain was affected. At the rehabilitative phase of your
3. The old woman complains of pain. John noticed that the knee is nursing care, you observe Mr. Rojas use a cane and you intervene if
reddened, warm to touch and swollen. John interprets that this signs you see him
and symptoms are likely related to
A. Moves the cane when the right leg is moved.
A. Infection
B. Leans on the cane when the right leg swings through.
B. Thrombophlebitis
C. keeps the cane 6 Inches out to the side of the right foot.
C. Inflammation
D. Holds the cane on the right side.
D. Degenerative disease
SITUATION: Alfred, a 40 year old construction worker developed
4. The old woman told John that she has osteoporosis; Arthur knew cough, night sweats and fever. He was brought to the nursing unit
that all of the following factors would contribute to osteoporosis except for diagnostic studies. He told the nurse he did not receive a BCG
vaccine during childhood
A. Hypothyroidism
9. The nurse performs a Mantoux Test. The nurse knows that Mantoux
B. End stage renal disease Test is also known as

C. Cushing’s Disease A. PPD

D. Taking Furosemide and Phenytoin. B. PDP

5. Martha, The old woman was now Immobilized and brought to the C. PDD
emergency room. The X-ray shows a fractured femur and pelvis. The
ER Nurse would carefully monitor Martha for which of the following D. DPP
sign and symptoms?
10. The nurse would inject the solution in what route?
A. Tachycardia and Hypotension
A. IM
B. Fever and Bradycardia
B. IV
C. ID B. 3 months

D. SC C. 4 months

11. The nurse notes that a positive result for Alfred is D. 5 months

A. 5 mm wheal 17. Which of the following drugs is UNLIKELY given to Mang Alfred
during the maintenance phase?
B. 5 mm Induration
A. Rifampicin
C. 10 mm Wheal
B. Isoniazid
D. 10 mm Induration
C. Ethambutol
12. The nurse told Alfred to come back after
D. Pyridoxine
A. a week
18. According to the DOH, the most hazardous period for development
B. 48 hours of clinical disease is during the first

C. 1 day A. 6-12 months after

D. 4 days B. 3-6 months after

13. Mang Alfred returns after the Mantoux Test. The test result read C. 1-2 months after
POSITIVE. What should be the nurse’s next action?
D. 2-4 weeks after
A. Call the Physician
19. This is the name of the program of the DOH to control TB in the
B. Notify the radiology dept. for CXR evaluation country

C. Isolate the patient A. DOTS

D. Order for a sputum exam B. National Tuberculosis Control Program

14. Why is Mantoux test not routinely done in the Philippines? C. Short Coursed Chemotherapy

A. It requires a highly skilled nurse to perform a Mantoux D. Expanded Program for Immunization
test
20. Susceptibility for the disease [ TB ] is increased markedly in those
B. The sputum culture is the gold standard of PTB with the following condition except
Diagnosis and it will definitively determine the extent
of the cavitary lesions A. 23 Year old athlete with diabetes insipidus

C. Chest X Ray Can diagnose the specific B. 23 Year old athlete taking long term Decadron therapy
microorganism responsible for the lesions and anabolic steroids

D. Almost all Filipinos will test positive for Mantoux C. 23 Year old athlete taking illegal drugs and abusing
Test substances

15. Mang Alfred is now a new TB patient with an active disease. What D. Undernourished and Underweight individual who
is his category according to the DOH? undergone gastrectomy

A. I 21. Direct sputum examination and Chest X ray of TB symptomatic is


in what level of prevention?
B. II
A. Primary
C. III
B. Secondary
D. IV
C. Tertiary
16. How long is the duration of the maintenance phase of his
treatment? D. Quarterly

A. 2 months
SITUATION: Michiel, A male patient diagnosed with colon cancer B. Slow down the irrigation
was newly put in colostomy.
C. Tell the client that cramping will subside and is
22. Michiel shows the BEST adaptation with the new colostomy if he normal
shows which of the following?
D. Notify the physician
A. Look at the ostomy site
28. The next day, the nurse will assess Michiel’s stoma. The nurse
B. Participate with the nurse in his daily ostomy care noticed that a prolapsed stoma is evident if she sees which of the
following?
C. Ask for leaflets and contact numbers of ostomy
support groups A. A sunken and hidden stoma

D. Talk about his ostomy openly to the nurse and friends B. A dusky and bluish stoma

23. The nurse plans to teach Michiel about colostomy irrigation. As the C. A narrow and flattened stoma
nurse prepares the materials needed, which of the following item
indicates that the nurse needs further instruction? D. Protruding stoma with swollen appearance

A. Plain NSS / Normal Saline 29. Michiel asked the nurse, what foods will help lessen the odor of his
colostomy. The nurse best response would be
B. K-Y Jelly
A. Eat eggs
C. Tap water
B. Eat cucumbers
D. Irrigation sleeve
C. Eat beet greens and parsley
24. The nurse should insert the colostomy tube for irrigation at
approximately D. Eat broccoli and spinach

A. 1-2 inches 30. The nurse will start to teach Michiel about the techniques for
colostomy irrigation. Which of the following should be included in the
B. 3-4 inches nurse’s teaching plan?

C. 6-8 inches A. Use 500 ml to 1,000 ml NSS

D. 12-18 inches B. Suspend the irrigant 45 cm above the stoma

25. The maximum height of irrigation solution for colostomy is C. Insert the cone 4 cm in the stoma

A. 5 inches D. If cramping occurs, slow the irrigation

B. 12 inches 31. The nurse knew that the normal color of Michiel’s stoma should be

C. 18 inches A. Brick Red

D. 24 inches B. Gray

26. Which of the following behavior of the client indicates the best C. Blue
initial step in learning to care for his colostomy?
D. Pale Pink
A. Ask to defer colostomy care to another individual
SITUATION: James, A 27 basketball player sustained inhalation
B. Promises he will begin to listen the next day burn that required him to have tracheostomy due to massive upper
airway edema.
C. Agrees to look at the colostomy
32. Wilma, His sister and a nurse is suctioning the tracheostomy tube
D. States that colostomy care is the function of the nurse of James. Which of the following, if made by Wilma indicates that she
while he is in the hospital is committing an error?

27. While irrigating the client’s colostomy, Michiel suddenly A. Hyperventilating James with 100% oxygen before and
complains of severe cramping. Initially, the nurse would after suctioning

A. Stop the irrigation by clamping the tube B. Instilling 3 to 5 ml normal saline to loosen up
secretion
C. Applying suction during catheter withdrawal A. James’ respiratory rate is 18

D. Suction the client every hour B. James’ Oxygen saturation is 91%

33. What size of suction catheter would Wilma use for James, who is C. There are frank blood suction from the tube
6 feet 5 inches in height and weighing approximately 145 lbs?
D. There are moderate amount of tracheobronchial
A. Fr. 5 secretions

B. Fr. 10 39. Wilma knew that the maximum time when suctioning James is

C. Fr. 12 A. 10 seconds

D. Fr. 18 B. 20 seconds

34. Wilma is using a portable suction unit at home, What is the amount C. 30 seconds
of suction required by James using this unit?
D. 45 seconds
A. 2-5 mmHg
SITUATION : Juan Miguel Lopez Zobel Ayala de Batumbakal was
B. 5-10 mmHg diagnosed with Acute Close Angle Glaucoma. He is being seen by
Nurse Jet.
C. 10-15 mmHg
40. What specific manifestation would nurse Jet see in Acute close
D. 20-25 mmHg angle glaucoma that she would not see in an open angle glaucoma?

35. If a Wall unit is used, What should be the suctioning pressure A. Loss of peripheral vision
required by James?
B. Irreversible vision loss
A. 50-95 mmHg
C. There is an increase in IOP
B. 95-110 mmHg
D. Pain
C. 100-120 mmHg
41. Nurse jet knew that Acute close angle glaucoma is caused by
D. 155-175 mmHg
A. Sudden blockage of the anterior angle by the base of
36. Wilma was shocked to see that the Tracheostomy was dislodged. the iris
Both the inner and outer cannulas was removed and left hanging on
James’ neck. What are the 2 equipment’s at james’ bedside that could B. Obstruction in trabecular meshwork
help Wilma deal with this situation?
C. Gradual increase of IOP
A. New set of tracheostomy tubes and Oxygen tank
D. An abrupt rise in IOP from 8 to 15 mmHg
B. Theophylline and Epinephrine
42. Nurse jet performed a TONOMETRY test to Mr. Batumbakal.
C. Obturator and Kelly clamp What does this test measures

D. Sterile saline dressing A. It measures the peripheral vision remaining on the


client
37. Which of the following method if used by Wilma will best assure
that the tracheostomy ties are not too tightly placed? B. Measures the Intra Ocular Pressure

A. Wilma places 2 fingers between the tie and neck C. Measures the Client’s Visual Acuity

B. The tracheotomy can be pulled slightly away from the D. Determines the Tone of the eye in response to the
neck sudden increase in IOP.

C. James’ neck veins are not engorged 43. The Nurse notices that Mr. Batumbakal cannot anymore determine
RED from BLUE. The nurse knew that which part of the eye is affected
D. Wilma measures the tie from the nose to the tip of the by this change?
earlobe and to the xiphoid process.
A. IRIS
38. Wilma knew that James have an adequate respiratory condition if
she notices that B. PUPIL
C. RODS [RETINA] D. Going out in the sun

D. CONES [RETINA] 50. Mr. Batumbakal has undergone eye angiography using an
Intravenous dye and fluoroscopy. What activity is contraindicated
44. Nurse Jet knows that Aqueous Humor is produce where? immediately after procedure?

A. In the sub arachnoid space of the meninges A. Reading newsprint

B. In the Lateral ventricles B. Lying down

C. In the Choroids C. Watching TV

D. In the Ciliary Body D. Listening to the music

45. Nurse Jet knows that the normal IOP is 51. If Mr. Batumbakal is receiving pilocarpine, what drug should
always be available in any case systemic toxicity occurs?
A. 8-21 mmHg
A. Atropine Sulfate
B. 2-7 mmHg
B. Pindolol [Visken]
C. 31-35 mmHg
C. Naloxone Hydrochloride [Narcan]
D. 15-30 mmHg
D. Mesoridazine Besylate [Serentil]
46. Nurse Jet wants to measure Mr. Batumbakal’s CN II Function.
What test would Nurse Jet implement to measure CN II’s Acuity? SITUATION : Wide knowledge about the human ear, it’s parts and
it’s functions will help a nurse assess and analyze changes in the
A. Slit lamp adult client’s health.

B. Snellen’s Chart 52. Nurse Anna is doing a caloric testing to his patient, Aida, a 55 year
old university professor who recently went into coma after being
mauled by her disgruntled 3rd year nursing students whom she gave a
C. Wood’s light failing mark. After instilling a warm water in the ear, Anna noticed a
rotary nystagmus towards the irrigated ear. What does this means?
D. Gonioscopy
A. Indicates a CN VIII Dysfunction
47. The Doctor orders pilocarpine. Nurse jet knows that the action of
this drug is to B. Abnormal

A. Contract the Ciliary muscle C. Normal

B. Relax the Ciliary muscle D. Inconclusive

C. Dilate the pupils 53. Ear drops are prescribed to an infant, The most appropriate method
to administer the ear drops is
D. Decrease production of Aqueous Humor
A. Pull the pinna up and back and direct the solution
48. The doctor orders timolol [timoptic]. Nurse jet knows that the towards the eardrum
action of this drug is
B. Pull the pinna down and back and direct the solution
A. Reduce production of CSF onto the wall of the canal

B. Reduce production of Aquesous Humor C. Pull the pinna down and back and direct the solution
towards the eardrum
C. Constrict the pupil
D. Pull the pinna up and back and direct the solution onto
D. Relaxes the Ciliary muscle the wall of the canal

49. When caring for Mr. Batumbakal, Jet teaches the client to avoid 54. Nurse Jenny is developing a plan of care for a patient with
Menieres disease. What is the priority nursing intervention in the plan
A. Watching large screen TVs of care for this particular patient?

B. Bending at the waist A. Air, Breathing, Circulation

C. Reading books B. Love and Belongingness


C. Food, Diet and Nutrition D. I should avoid air travel for a while

D. Safety 60. Nurse Oca will do a caloric testing to a client who sustained a blunt
injury in the head. He instilled a cold water in the client’s right ear and
55. After mastoidectomy, Nurse John should be aware that the cranial he noticed that nystagmus occurred towards the left ear. What does this
nerve that is usually damage after this procedure is finding indicates?

A. CN I A. Indicating a Cranial Nerve VIII Dysfunction

B. CN II B. The test should be repeated again because the result is


vague
C. CN VII
C. This is Grossly abnormal and should be reported to the
D. CN VI neurosurgeon

56. The physician orders the following for the client with Menieres D. This indicates an intact and working vestibular branch
disease. Which of the following should the nurse question? of CN VIII

A. Dipenhydramine [Benadryl] 61. A client with Cataract is about to undergo surgery. Nurse Oca is
preparing plan of care. Which of the following nursing diagnosis is
most appropriate to address the long term need of this type of patient?
B. Atropine sulfate
A. Anxiety R/T to the operation and its outcome
C. Out of bed activities and ambulation
B. Sensory perceptual alteration R/T Lens extraction and
D. Diazepam [Valium]
replacement
57. Nurse Anna is giving dietary instruction to a client with Menieres
C. Knowledge deficit R/T the pre operative and post
disease. Which statement if made by the client indicates that the
operative self care
teaching has been successful?
D. Body Image disturbance R/T the eye packing after
A. I will try to eat foods that are low in sodium and limit
surgery
my fluid intake
62. Nurse Joseph is performing a WEBERS TEST. He placed the
B. I must drink atleast 3,000 ml of fluids per day
tuning fork in the patients forehead after tapping it onto his knee. The
client states that the fork is louder in the LEFT EAR. Which of the
C. I will try to follow a 50% carbohydrate, 30% fat and following is a correct conclusion for nurse Josph to make?
20% protein diet
A. He might have a sensory hearing loss in the left ear
D. I will not eat turnips, red meat and raddish
B. Conductive hearing loss is possible in the right ear
58. Peachy was rushed by his father, Steven into the hospital
admission. Peachy is complaining of something buzzing into her ears.
C. He might have a sensory hearing loss in the right hear,
Nurse Joemar assessed peachy and found out It was an insect. What
and/or a conductive hearing loss in the left ear.
should be the first thing that Nurse Joemar should try to remove the
insect out from peachy’s ear?
D. He might have a conductive hearing loss in the right
ear, and/or a sensory hearing loss in the left ear.
A. Use a flashlight to coax the insect out of peachy’s ear
63. Aling myrna has Menieres disease. What typical dietary
B. Instill an antibiotic ear drops
prescription would nurse Oca expect the doctor to prescribe?
C. Irrigate the ear
A. A low sodium , high fluid intake
D. Pick out the insect using a sterile clean forceps
B. A high calorie, high protein dietary intake
59. Following an ear surgery, which statement if heard by Nurse Oca
C. low fat, low sodium and high calorie intake
from the patient indicates a correct understanding of the post operative
instructions?
D. low sodium and restricted fluid intake
A. Activities are resumed within 5 days
SITUATION : [ From DEC 1991 NLE ] A 45 year old male
construction worker was admitted to a tertiary hospital for
B. I will make sure that I will clean my hair and face to
prevent infection incessant vomiting. Assessment disclosed: weak rapid pulse, acute
weight loss of .5kg, furrows in his tongue, slow flattening of the skin
was noted when the nurse released her pinch.
C. I will use straw for drinking
Temperature: 35.8 C , BUN Creatinine ratio : 10 : 1, He also
complains for postural hypotension. There was no infection.
SITUATION: A 65 year old woman was admitted for Parkinson’s
64. Which of the following is the appropriate nursing diagnosis? Disease. The charge nurse is going to make an initial assessment.

A. Fluid volume deficit R/T furrow tongue 69. Which of the following is a characteristic of a patient with
advanced Parkinson’s disease?
B. Fluid volume deficit R/T uncontrolled vomiting
A. Disturbed vision
C. Dehydration R/T subnormal body temperature
B. Forgetfulness
D. Dehydration R/T incessant vomiting
C. Mask like facial expression
65. Approximately how much fluid is lost in acute weight loss of .5kg?
D. Muscle atrophy
A. 50 ml
70. The onset of Parkinson’s disease is between 50-60 years old. This
B. 750 ml disorder is caused by

C. 500 ml A. Injurious chemical substances

D. 75 ml B. Hereditary factors

66. Postural Hypotension is C. Death of brain cells due to old age

A. A drop in systolic pressure less than 10 mmHg when D. Impairment of dopamine producing cells in the brain
patient changes position from lying to sitting.
71. The patient was prescribed with levodopa. What is the action of
B. A drop in systolic pressure greater than 10 mmHg this drug?
when patient changes position from lying to sitting
A. Increase dopamine availability
C. A drop in diastolic pressure less than 10 mmHg when
patient changes position from lying to sitting B. Activates dopaminergic receptors in the basal ganglia

D. A drop in diastolic pressure greater than 10 mmHg C. Decrease acetylcholine availability


when patient changes position from lying to sitting
D. Release dopamine and other catecholamine from
67. Which of the following measures will not help correct the patient’s neurological storage sites
condition
72. You are discussing with the dietician what food to avoid with
A. Offer large amount of oral fluid intake to replace fluid patients taking levodopa?
lost
A. Vitamin C rich food
B. Give enteral or parenteral fluid
B. Vitamin E rich food
C. Frequent oral care
C. Thiamine rich food
D. Give small volumes of fluid at frequent interval
D. Vitamin B6 rich food
68. After nursing intervention, you will expect the patient to have
73. One day, the patient complained of difficulty in walking. Your
1. Maintain body temperature at 36.5 C response would be

2. Exhibit return of BP and Pulse to normal A. You will need a cane for support

3. Manifest normal skin turgor of skin and tongue B. Walk erect with eyes on horizon

4. Drinks fluids as prescribed C. I’ll get you a wheelchair

A. 1,3 D. Don’t force yourself to walk

B. 2,4 SITUATION: Mr. Dela Isla, a client with early Dementia exhibits
thought process disturbances.
C. 1,3,4
74. The nurse will assess a loss of ability in which of the following
D. 2,3,4 areas?
A. Balance C. Reduces secretion of the glandular organ of the body

B. Judgment D. Stimulate peristalsis for treatment of constipation and


obstruction
C. Speech
80. What should the nurse caution the client when using this
D. Endurance medication

75. Mr. Dela Isla said he cannot comprehend what the nurse was A. Avoid hazardous activities like driving, operating
saying. He suffers from: machineries etc.

A. Insomnia B. Take the drug on empty stomach

B. Aphraxia C. Take with a full glass of water in treatment of


Ulcerative colitis
C. Agnosia
D. I must take double dose if I missed the previous dose
D. Aphasia
81. Which of the following drugs are not compatible when taking
76. The nurse is aware that in communicating with an elderly client, Probanthine?
the nurse will
A. Caffeine
A. Lean and shout at the ear of the client
B. NSAID
B. Open mouth wide while talking to the client
C. Acetaminophen
C. Use a low-pitched voice
D. Alcohol
D. Use a medium-pitched voice
82. What should the nurse tell clients when taking Probanthine?
77. As the nurse talks to the daughter of Mr. Dela Isla, which of the
following statement of the daughter will require the nurse to give A. Avoid hot weathers to prevent heat strokes
further teaching?
B. Never swim on a chlorinated pool
A. I know the hallucinations are parts of the disease
C. Make sure you limit your fluid intake to 1L a day
B. I told her she is wrong and I explained to her what is
right D. Avoid cold weathers to prevent hypothermia

C. I help her do some tasks he cannot do for himself 83. Which of the following disease would Probanthine exert the much
needed action for control or treatment of the disorder?
D. Ill turn off the TV when we go to another room
A. Urinary retention
78. Which of the following is most important discharge teaching for
Mr. Dela Isla B. Peptic Ulcer Disease

A. Emergency Numbers C. Ulcerative Colitis

B. Drug Compliance D. Glaucoma

C. Relaxation technique SITUATION : Mr. Franco, 70 years old, suddenly could not lift his
spoons nor speak at breakfast. He was rushed to the hospital
D. Dietary prescription unconscious. His diagnosis was CVA.

SITUATION : Knowledge of the drug PROPANTHELINE BROMIDE 84. Which of the following is the most important assessment during
[Probanthine] Is necessary in treatment of various disorders. the acute stage of an unconscious patient like Mr. Franco?

79. What is the action of this drug? A. Level of awareness and response to pain

A. Increases glandular secretion for clients affected with B. Papillary reflexes and response to sensory stimuli
cystic fibrosis
C. Coherence and sense of hearing
B. Dissolve blockage of the urinary tract due to
obstruction of cystine stones D. Patency of airway and adequacy of respiration
85. Considering Mr. Franco’s conditions, which of the following is 90. Stat appendectomy was indicated. Pre op care would include all of
most important to include in preparing Franco’s bedside equipment? the following except?

A. Hand bell and extra bed linen A. Consent signed by the father

B. Sandbag and trochanter rolls B. Enema STAT

C. Footboard and splint C. Skin prep of the area including the pubis

D. Suction machine and gloves D. Remove the jewelries

86. What is the rationale for giving Mr. Franco frequent mouth care? 91. Pre-anesthetic med of Demerol and atrophine sulfate were ordered
to :
A. He will be thirsty considering that he is doesn’t drink
enough fluids A. Allay anxiety and apprehension

B. To remove dried blood when tongue is bitten during a B. Reduce pain


seizure
C. Prevent vomiting
C. The tactile stimulation during mouth care will hasten
return to consciousness D. Relax abdominal muscle

D. Mouth breathing is used by comatose patient and it’ll 92. Common anesthesia for appendectomy is
cause oral mucosa dying and cracking.
A. Spinal
87. One of the complications of prolonged bed rest is decubitus ulcer.
Which of the following can best prevent its occurrence? B. General

A. Massage reddened areas with lotion or oils C. Caudal

B. Turn frequently every 2 hours D. Hypnosis

C. Use special water mattress 93. Post op care for appendectomy include the following except

D. Keep skin clean and dry A. Early ambulation

88. If Mr. Franco’s Right side is weak, What should be the most B. Diet as tolerated after fully conscious
accurate analysis by the nurse?
C. Nasogastric tube connect to suction
A. Expressive aphasia is prominent on clients with right
sided weakness
D. Deep breathing and leg exercise
B. The affected lobe in the patient is the Right lobe
94. Peritonitis may occur in ruptured appendix and may cause serious
problems which are
C. The client will have problems in judging distance and
proprioception
1. Hypovolemia, electrolyte imbalance
D. Clients orientation to time and space will be much
2. Elevated temperature, weakness and diaphoresis
affected
3. Nausea and vomiting, rigidity of the abdominal wall
SITUATION : a 20 year old college student was rushed to the ER of
PGH after he fainted during their ROTC drill. Complained of severe
right iliac pain. Upon palpation of his abdomen, Ernie jerks even on 4. Pallor and eventually shock
slight pressure. Blood test was ordered. Diagnosis is acute
appendicitis. A. 1 and 2

89. Which result of the lab test will be significant to the diagnosis? B. 2 and 3

A. RBC : 4.5 TO 5 Million / cu. mm. C. 1,2,3

B. Hgb : 13 to 14 gm/dl. D. All of the above

C. Platelets : 250,000 to 500,000 cu.mm. 95. If after surgery the patient’s abdomen becomes distended and no
bowel sounds appreciated, what would be the most suspected
D. WBC : 12,000 to 13,000/cu.mm complication?
A. Intussusception Answers

B. Paralytic Ileus 1. D. Move the person to a safer place.

C. Hemorrhage 2. D. Shortened, Adducted and Externally Rotated.

D. Ruptured colon 3. C. Inflammation

96. NGT was connected to suction. In caring for the patient with NGT, 4. A. Hypothyroidism
the nurse must
5. A. Tachycardia and Hypotension
A. Irrigate the tube with saline as ordered
6. B. On his left hand, because of reciprocal motion.
B. Use sterile technique in irrigating the tube
7. c. 6 Inches at the lateral side of the foot.
C. advance the tube every hour to avoid kinks
8. A. Moves the cane when the right leg is moved.
D. Offer some ice chips to wet lips
9. A. PPD
97. When do you think the NGT tube be removed?
10. C. ID
A. When patient requests for it
11. D. 10 mm Induration
B. Abdomen is soft and patient asks for water
12. B. 48 hours
C. Abdomen is soft and flatus has been expelled
13. A. Call the Physician
D. B and C only
14. D. Almost all Filipinos will test positive for
Situation: Amanda is suffering from chronic arteriosclerosis Brain Mantoux Test
syndrome she fell while getting out of the bed one morning and was
brought to the hospital, and she was diagnosed to have
15. A. I
cerebrovascular thrombosis thus transferred to a nursing home.
16. C. 4 months
98. What do you call a STROKE that manifests a bizarre behavior?
17. C. Ethambutol
A. Inorganic Stroke
18. A. 6-12 months after
B. Inorganic Psychoses
19. B. National Tuberculosis Control Program
C. Organic Stroke

D. Organic Psychoses 20. A. 23 Year old athlete with diabetes insipidus

99. The main difference between chronic and organic brain syndrome 21. B. Secondary
is that the former
22. B. Participate with the nurse in his daily ostomy
A. Occurs suddenly and reversible care

B. Is progressive and reversible 23. A. Plain NSS / Normal Saline

C. tends to be progressive and irreversible 24. B. 3-4 inches

D. Occurs suddenly and irreversible 25. C. 18 inches

100. Which behavior results from organic psychoses? 26. C. Agrees to look at the colostomy

A. Memory deficit 27. A. Stop the irrigation by clamping the tube

B. Disorientation 28. D. Protruding stoma with swollen appearance

C. Impaired Judgement 29. C. Eat beet greens and parsley

D. Inappropriate affect 30. B. Suspend the irrigant 45 cm above the stoma


31. A. Brick Red 61. B. Sensory perceptual alteration R/T Lens
extraction and replacement
32. D. Suction the client every hour
62. C. He might have a sensory hearing loss in the right
33. D. Fr. 18 hear, and/or a conductive hearing loss in the left
ear.
34. C. 10-15 mmHg
63. D. low sodium and restricted fluid intake
35. C. 100-120 mmHg
64. B. Fluid volume deficit R/T uncontrolled vomiting
36. C. Obturator and Kelly clamp
65. C. 500 ml
37. A. Wilma places 2 fingers between the tie and neck
66. B. A drop in systolic pressure greater than 10
38. A. James’ respiratory rate is 18 mmHg when patient changes position from lying to
sitting
39. A. 10 seconds
67. A. Offer large amount of oral fluid intake to replace
40. D. Pain fluid lost

41. A. Sudden blockage of the anterior angle by the 68. D. 2,3,4


base of the iris
69. C. Mask like facial expression
42. B. Measures the Intra Ocular Pressure
70. D. Impairment of dopamine producing cells in the
43. D. CONES [RETINA] brain

44. D. In the Ciliary Body 71. A. Increase dopamine availability

45. A. 8-21 mmHg 72. D. Vitamin B6 rich food

46. B. Snellen’s Chart 73. A. You will need a cane for support

47. A. Contract the Ciliary muscle 74. B. Judgment

48. B. Reduce production of Aquesous Humor 75. D. Aphasia

49. B. Bending at the waist 76. D. Use a medium-pitched voice

50. A. Reading newsprint 77. B. I told her she is wrong and I explained to her
what is right
51. A. Atropine Sulfate
78. B. Drug Compliance
52. C. Normal
79. C. Reduces secretion of the glandular organ of the
body
53. B. Pull the pinna down and back and direct the
solution onto the wall of the canal
80. A. Avoid hazardous activities like driving,
operating machineries etc.
54. D. Safety
81. D. Alcohol
55. C. CN VII
82. A. Avoid hot weathers to prevent heat strokes
56. C. Out of bed activities and ambulation
83. B. Peptic Ulcer Disease
57. A. I will try to eat foods that are low in sodium and
limit my fluid intake
84. D. Patency of airway and adequacy of respiration
58. A. Use a flashlight to coax the insect out of peachy’s
ear 85. D. Suction machine and gloves

59. D. I should avoid air travel for a while 86. D. Mouth breathing is used by comatose patient
and it’ll cause oral mucosa dying and cracking.
60. D. This indicates an intact and working vestibular
branch of CN VIII 87. B. Turn frequently every 2 hours
88. A. Expressive aphasia is prominent on clients with C. clay-colored stools
right sided weakness
D. widened pulse pressure
89. D. WBC : 12,000 to 13,000/cu.mm
4. A client is hospitalized with a diagnosis of chronic
90. B. Enema STAT glomerulonephritis. The client mentions that she likes salty foods. The
nurse should warn her to avoid foods containing sodium because:
91. A. Allay anxiety and apprehension
A. reducing sodium promotes urea nitrogen excretion
92. A. Spinal
B. reducing sodium improves her glomerular filtration
93. B. Diet as tolerated after fully conscious rate

94. D. All of the above C. reducing sodium increases potassium absorption

95. B. Paralytic Ileus D. reducing sodium decreases edema

96. A. Irrigate the tube with saline as ordered 5. The nurse is caring for a client with a cerebral injury that impaired
his speech and hearing. Most likely, the client has experienced damage
to the:
97. C. Abdomen is soft and flatus has been expelled
A. frontal lobe
98. D. Organic Psychoses
B. parietal lobe
99. C. tends to be progressive and irreversible
C. occipital lobe
100. B. Disorientation
D. temporal lobe

6. The nurse is assessing a postcraniotomy client and finds the urine


Medical Surgical Nursing Exam 2 output from a catheter is 1500 ml for the 1st hour and the same for the
2nd hour. The nurse should suspect:
1. After a cerebrovascular accident, a 75 yr old client is admitted to the
health care facility. The client has left-sided weakness and an absent A. Cushing’s syndrome
gag reflex. He’s incontinent and has a tarry stool. His blood pressure
is 90/50 mm Hg, and his hemoglobin is 10 g/dl. Which of the following B. Diabetes mellitus
is a priority for this client?
C. Adrenal crisis
A. checking stools for occult blood
D. Diabetes insipidus
B. performing range-of-motion exercises to the left side
7. The nurse is providing postprocedure care for a client who
C. keeping skin clean and dry underwent percutaneous lithotripsy. In this procedure, an ultrasonic
probe inserted through a nephrostomy tube into the renal pelvis
D. elevating the head of the bed to 30 degrees generates ultra-high-frequency sound waves to shatter renal calculi.
The nurse should instruct the client to:
2. The nurse is caring for a client with a colostomy. The client tells the
nurse that he makes small pin holes in the drainage bag to help relieve A. limit oral fluid intake for 1 to 2 weeks
gas. The nurse should teach him that this action:
B. report the presence of fine, sandlike particles through
A. destroys the odor-proof seal the nephrostomy tube.

B. wont affect the colostomy system C. Notify the physician about cloudy or foul smelling
urine
C. is appropriate for relieving the gas in a colostomy
system D. Report bright pink urine within 24 hours after the
procedure
D. destroys the moisture barrier seal
8. A client with a serum glucose level of 618 mg/dl is admitted to the
3. When assessing the client with celiac disease, the nurse can expect facility. He’s awake and oriented, has hot dry skin, and has the
following vital signs: temperature of 100.6º F (38.1º C), heart rate of
to find which of the following?
116 beats/minute, and blood pressure of 108/70 mm Hg. Based on
these assessment findings, which nursing diagnosis takes the highest
A. steatorrhea priority?

B. jaundiced sclerae
A. deficient fluid volume related to osmotic diuresis A. basilar

B. decreased cardiac output related to elevated heart rate B. temporal

C. imbalanced nutrition: Less than body requirements C. occipital


related to insulin deficiency
D. parietal
D. ineffective thermoregulation related to dehydration
14. A male client should be taught about testicular examinations:
9. Capillary glucose monitoring is being performed every 4 hours for
a client diagnosed with diabetic ketoacidosis. Insulin is administered A. when sexual activity starts
using a scale of regular insulin according to glucose results. At 2 p.m.,
the client has a capillary glucose level of 250 mg/dl for which he B. after age 60
receives 8 U of regular insulin. The
nurse should expect the dose’s:
C. after age 40
A. onset to be at 2 p.m. and its peak at 3 p.m.
D. before age 20
B. onset to be at 2:15 p.m. and its peak at 3 p.m.
15. Before weaning a client from a ventilator, which assessment
parameter is most important for the nurse to review?
C. onset to be at 2:30 p.m. and its peak at 4 p.m.
A. fluid intake for the last 24 hours
D. onset to be at 4 p.m. and its peak at 6 p.m.
B. baseline arterial blood gas (ABG) levels
10. A client with a head injury is being monitored for increased
intracranial pressure (ICP). His blood pressure is 90/60 mmHG and the
C. prior outcomes of weaning
ICP is 18 mmHg; therefore his cerebral perfusion pressure (CPP) is:
D. electrocardiogram (ECG) results
A. 52 mm Hg
16. The nurse is speaking to a group of women about early detection
B. 88 mm Hg
of breast cancer. The average age of the women in the group is 47.
Following the American Cancer Society (ACS) guidelines, the nurse
C. 48 mm Hg should recommend that the women:

D. 68 mm Hg A. perform breast self-examination annually

11. A 52 yr-old female tells the nurse that she has found a painless B. have a mammogram annually
lump in her right breast during her monthly self-examination. Which
assessment finding would strongly suggest that this client’s lump is
C. have a hormonal receptor assay annually
cancerous?
D. have a physician conduct a clinical evaluation every 2
A. eversion of the right nipple and a mobile mass
years
B. nonmobile mass with irregular edges
17. When caring for a client with esophageal varices, the nurse knows
that bleeding in this disorder usually stems from:
C. mobile mass that is oft and easily delineated
A. esophageal perforation
D. nonpalpable right axillary lymph nodes
B. pulmonary hypertension
12. A Client is scheduled to have a descending colostomy. He’s very
anxious and has many questions regarding the surgical procedure, care
C. portal hypertension
of stoma, and lifestyle changes. It would be most appropriate for the
nurse to make a referral to which member of the health care team?
D. peptic ulcers
A. Social worker
18. A 49-yer-old client was admitted for surgical repair of a Colles’
fracture. An external fixator was placed during surgery. The surgeon
B. registered dietician
explains that this method of repair:
C. occupational therapist
A. has very low complication rate
D. enterostomal nurse therapist
B. maintains reduction and overall hand function
13. Ottorrhea and rhinorrhea are most commonly seen with which type
C. is less bothersome than a cast
of skull fracture?
D. is best for older people
19. A client is hospitalized with a diagnosis of chronic renal failure. C. A pulse that isn’t easily obliterated.
An arteriovenous fistula was created in his left arm for hemodialysis.
When preparing the client for discharge, the nurse should reinforce D. Neck vein distention
which dietary instruction?
24. The nurse is teaching a client with a history of atherosclerosis. To
A. “Be sure to eat meat at every meal.” decrease the risk of atherosclerosis, the nurse should encourage the
client to:
B. “Monitor your fruit intake and eat plenty of bananas.”
A. Avoid focusing on his weight.
C. “Restrict your salt intake.”
B. Increase his activity level.
D. “Drink plenty of fluids.”
C. Follow a regular diet.
20. The nurse is caring for a client who has just had a modified radical
mastectomy with immediate reconstruction. She’s in her 30s and has D. Continue leading a high-stress lifestyle.
tow children. Although she’s worried about her future, she seems to be
adjusting well to her diagnosis. What should the nurse do to support 25. For a client newly diagnosed with radiationinduced
her coping? thrombocytopenia, the nurse should include which intervention in the
plan of care?
A. Tell the client’s spouse or partner to be supportive
while she recovers. A. Administer aspirin if the temperature exceeds 38.8º C.

B. Encourage the client to proceed with the next phase of B. Inspect the skin for petechiae once every shift.
treatment.
C. Provide for frequent periods of rest.
C. Recommend that the client remain cheerful for the
sake of her children.
D. Place the client in strict isolation.
D. Refer the client to the American Cancer Society’s
26. A client is chronically short of breath and yet has normal lung
Reach for Recovery program or another support
ventilation, clear lungs, and an arterial oxygen saturation (SaO2) 96%
program.
or better. The client most likely has:
21. A 21 year-old male has been seen in the clinic for a thickening in
A. poor peripheral perfusion
his right testicle. The physician ordered a human chorionic
gonadotropin (HCG) level. The nurse’s explanation to the client should
include the fact that: B. a possible Hematologic problem

A. The test will evaluate prostatic function. C. a psychosomatic disorder

B. The test was ordered to identify the site of a possible D. left-sided heart failure
infection.
27. For a client in addisonian crisis, it would be very risky for a nurse
C. The test was ordered because clients who have to administer:
testicular cancer has elevated levels of HCG.
A. potassium chloride
D. The test was ordered to evaluate the testosterone level.
B. normal saline solution
22. A client is receiving captopril (Capoten) for heart failure. The nurse
should notify the physician that the medication therapy is ineffective C. hydrocortisone
if an assessment reveals:
D. fludrocortisone
A. A skin rash.
28. The nurse is reviewing the laboratory report of a client who
B. Peripheral edema. underwent a bone marrow biopsy. The finding that would most
strongly support a diagnosis of acute leukemia is the existence of a
C. A dry cough. large number of immature:

D. Postural hypotension. A. lymphocytes

23. Which assessment finding indicates dehydration? B. thrombocytes

A. Tenting of chest skin when pinched. C. reticulocytes

B. Rapid filling of hand veins. D. leukocytes


29. The nurse is performing wound care on a foot ulcer in a client with B. Imbalanced nutrition: less than body requirements
type 1 diabetes mellitus. Which technique demonstrates surgical related to hypermetabolic state
asepsis?
C. Deficient fluid volume related to osmotic diuresis
A. Putting on sterile gloves then opening a container of induced by hypernatremia
sterile saline.
D. Imbalanced nutrition: less than body requirements
B. Cleaning the wound with a circular motion, moving related to catabolic effects of insulin deficiency
from outer circles toward the center.
34. A 20 yr-old woman has just been diagnosed with Crohn’s disease.
C. Changing the sterile field after sterile water is spilled She has lost 10 lb (4.5 kg) and has cramps and occasional diarrhea. The
on it. nurse should include which of the following when doing a nutritional
assessment?
D. Placing a sterile dressing ½” (1.3 cm) from the edge
of the sterile field. A. Let the client eat as desired during the hospitalization.

30. A client with a forceful, pounding heartbeat is diagnosed with B. Weight the client daily.
mitral valve prolapse. This client should avoid which of the following?
C. Ask the client to list what she eats during a typical day.
A. high volumes of fluid intake
D. Place the client on I & O status and draw blood for
B. aerobic exercise programs electrolyte levels.

C. caffeine-containing products 35. When instructions should be included in the discharge teaching
plan for a client after thyroidectomy for Grave’s disease?
D. foods rich in protein
A. Keep an accurate record of intake and output.
31. A client with a history of hypertension is diagnosed with primary
hyperaldosteronism. This diagnosis indicates that the client’s B. Use nasal desmopressin acetate DDAVP).
hypertension is caused by excessive hormone secretion from which
organ? C. Be sure to get regulate follow-up care.

A. adrenal cortex D. Be sure to exercise to improve cardiovascular fitness.

B. pancreas 36. A client comes to the emergency department with chest pain,
dyspnea, and an irregular heartbeat. An electrocardiogram shows a
C. adrenal medulla heart rate of 110 beats/minute (sinus tachycardia) with frequent
premature ventricular contractions. Shortly after admission, the client
D. parathyroid has ventricular tachycardia and becomes unresponsive. After
successful resuscitation, the client is taken to the intensive care unit.
32. A client has a medical history of rheumatic fever, type 1 (insulin Which nursing diagnosis is appropriate at this time?
dependent) diabetes mellitus, hypertension, pernicious anemia, and
appendectomy. She’s admitted to the hospital and undergoes mitral A. Deficient knowledge related to interventions used to
valve replacement surgery. After discharge, the client is scheduled for treat acute illness
a tooth extraction. Which history finding is a major risk factor for
infective endocarditis? B. Impaired physical mobility related to complete bed
rest
A. appendectomy
C. Social isolation related to restricted visiting hours in
B. pernicious anemia the intensive care unit

C. diabetes mellitus D. Anxiety related to the threat of death

D. valve replacement 37. A client is admitted to the health care facility with active
tuberculosis. The nurse should include which intervention in the plan
33. A 62 yr-old client diagnosed with pyelonephritis and possible of care?
septicemia has had five urinary tract infections over the past two years.
She’s fatigued from lack of sleep; urinates frequently, even during the A. Putting on a mask when entering the client’s room.
night; and has lost weight recently. Test reveal the following: sodium
level 152 mEq/L, osmolarity 340 mOsm/L, glucose level 125 mg/dl, B. Instructing the client to wear a mask at all times
and potassium level 3.8 mEq/L. which of the following nursing
diagnoses is most appropriate for this client? C. Wearing a gown and gloves when providing direct
care
A. Deficient fluid volume related to inability to conserve
water D. Keeping the door to the client’s room open to observe
the client
38. The nurse is caring for a client who underwent a subtotal A. “You seem angry. Would you like to talk about it?”
gastrectomy 24 hours earlier. The client has a nasogastric (NG) tube.
The nurse should: B. “Calm down. You know that stress will make your
symptoms worse.”
A. Apply suction to the NG tube every hour.
C. “Would you like to talk about the problem with the
B. Clamp the NG tube if the client complains of nausea. nursing supervisor?”

C. Irrigate the NG tube gently with normal saline D. “I can see you’re angry. I’ll come back when you’ve
solution. calmed down.”

D. Reposition the NG tube if pulled out. 44. On a routine visit to the physician, a client with chronic arterial
occlusive disease reports stopping smoking after 34 years. To relive
39. Which statement about fluid replacement is accurate for a client symptoms of intermittent claudication, a condition associated with
with hyperosmolar hyperglycemic nonketotic syndrome (HHNS)? chronic arterial occlusive disease, the nurse should recommend which
additional measure?
A. administer 2 to 3 L of IV fluid rapidly
A. Taking daily walks.
B. administer 6 L of IV fluid over the first 24 hours
B. Engaging in anaerobic exercise.
C. administer a dextrose solution containing normal
saline solution C. Reducing daily fat intake to less than 45% of total
calories
D. administer IV fluid slowly to prevent circulatory
overload and collapse D. Avoiding foods that increase levels of highdensity
lipoproteins (HDLs)
40. Which of the following is an adverse reaction to glipizide
(Glucotrol)? 45. A physician orders gastric decompression for a client with small
bowel obstruction. The nurse should plan for the suction to be:
A. headache
A. low pressure and intermittent
B. constipation
B. low pressure and continuous
C. hypotension
C. high pressure and continuous
D. photosensitivity
D. high pressure and intermittent
41. The nurse is caring for four clients on a stepdown intensive care
unit. The client at the highest risk for developing nosocomial 46. Which nursing diagnosis is most appropriate for an elderly client
pneumonia is the one who: with osteoarthritis?

A. has a respiratory infection A. Risk for injury

B. is intubated and on a ventilator B. Impaired urinary elimination

C. has pleural chest tubes C. Ineffective breathing pattern

D. is receiving feedings through a jejunostomy tube D. Imbalanced nutrition: less than body requirements

42. The nurse is teaching a client with chronic bronchitis about 47. Parathyroid hormone (PTH) has which effects on the kidney?
breathing exercises. Which of the following should the nurse include
in the teaching? A. Stimulation of calcium reabsorption and phosphate
excretion
A. Make inhalation longer than exhalation.
B. Stimulation of phosphate reabsorption and calcium
B. Exhale through an open mouth. excretion

C. Use diaphragmatic breathing. C. Increased absorption of vit D and excretion of vit E

D. Use chest breathing. D. Increased absorption of vit E and excretion of Vit D

43. A client is admitted to the hospital with an exacerbation of her 48. A visiting nurse is performing home assessment for a 59-yr old
chronic systemic lupus erythematosus (SLE). She gets angry when her man recently discharged after hip replacement surgery. Which home
call bell isn’t answered immediately. The most appropriate response to assessment finding warrants health promotion teaching from the
her would be: nurse?
A. A bathroom with grab bars for the tub and toilet 53. A client with type 1 diabetes mellitus has been on a regimen of
multiple daily injection therapy. He’s being converted to continuous
B. Items stored in the kitchen so that reaching up and subcutaneous insulin therapy. While teaching the client bout
bending down aren’t necessary continuous subcutaneous insulin therapy, the nurse would be accurate
in telling him the regimen includes the use of:
C. Many small, unsecured area rugs
A. intermediate and long-acting insulins
D. Sufficient stairwell lighting, with switches to the top
and bottom of the stairs B. short and long-acting insulins

49. A client with autoimmune thrombocytopenia and a platelet count C. short-acting only
of 800/uL develops epistaxis and melena. Treatment with
corticosteroids and immunoglobulins has been unsuccessful, and the D. short and intermediate-acting insulins
physician recommends a splenectomy. The client states, “I don’t need
surgery—this will go away on its own.” In considering her response to 54. a client who recently had a cerebrovascular accident requires a cane
the client, the nurse must depend on the ethical principle of: to ambulate. When teaching about cane use, the rationale for holding a
cane on the uninvolved side is to:
A. beneficence
A. prevent leaning
B. autonomy
B. distribute weight away from the involved side
C. advocacy
C. maintain stride length
D. justice
D. prevent edema
50. Which of the following is t he most critical intervention needed for
a client with myxedema coma? 55. A client with a history of an anterior wall myocardial infarction is
being transferred from the coronary care unit (CCU) to the cardiac
A. Administering and oral dose of levothyroxine stepdown unit (CSU). While giving report to the CSU nurse, the CCU
(Synthroid) nurse says, “His pulmonary artery wedge pressures have been in the
high normal range.” The CSU nurse should be especially observant
B. Warming the client with a warming blanket for:

C. Measuring and recording accurate intake and output A. hypertension

D. Maintaining a patent airway B. high urine output

51. Because diet and exercise have failed to control a 63 yr-old client’s C. dry mucous membranes
blood glucose level, the client is prescribed glipizide (Glucotrol). After
oral administration, the onset of action is: D. pulmonary crackles

A. 15 to 30 minutes 56. The nurse is caring for a client with a fractures hip. The client is
combative, confused, and trying to get out of bed. The nurse should:
B. 30 to 60 minutes
A. leave the client and get help
C. 1 to 1 ½ hours
B. obtain a physician’s order to restrain the client
D. 2 to 3 hours
C. read the facility’s policy on restraints
52. A client with pneumonia is receiving supplemental oxygen, 2
L/min via nasal cannula. The client’s history includes chronic D. order soft restraints from the storeroom
obstructive pulmonary disease (COPD) and coronary artery disease.
Because of these findings, the nurse closely monitors the oxygen flow 57. For the first 72 hours after thyroidectomy surgery, the nurse would
and the client’s respiratory status. Which complication may arise if the assess the client for Chvostek’s sign and Trousseau’s sign because they
client receives a high oxygen concentration? indicate which of the following?

A. Apnea A. hypocalcemia

B. Anginal pain B. hypercalcemia

C. Respiratory alkalosis C. hypokalemia

D. Metabolic acidosis D. Hyperkalemia


58. In a client with enteritis and frequent diarrhea, the nurse should C. gathering a list of persons with whom she has had
anticipate an acidbase imbalance of: recent contact

A. respiratory acidosis D. advising her to begin prophylactic therapy with


isoniazid (INH)
B. respiratory alkalosis
64. The nurse is caring for a client who ahs had an above the knee
C. metabolic acidosis amputation. The client refuses to look at the stump. When the nurse
attempts to speak with the client about his surgery, he tells the nurse
D. metabolic alkalosis that he doesn’t wish to discuss it. The client also refuses to have his
family visit. The nursing diagnosis that best describes the client’s
problem is:
59. When caring for a client with the nursing diagnosis Impaired
swallowing related to neuromuscular impairment, the nurse should:
A. Hopelessness
A. position the client in a supine position
B. Powerlessness
B. elevate the head of the bed 90 degrees during meals
C. Disturbed body image
C. encourage the client to remove dentures
D. Fear
D. encourage thin liquids for dietary intake
65. A client with three children who is still I the child bearing years is
admitted for surgical repair of a prolapsed bladder. The nurse would
60. A nurse is caring for a client who has a tracheostomy and
find that the client understood the surgeon’s preoperative teaching
temperature of 39º C. which intervention will most likely lower the
when the client states:
client’s arterial blood oxygen saturation?
A. “If I should become pregnant again, the child would
A. Endotracheal suctioning
be delivered by cesarean delivery.”
B. Encouragement of coughing
B. “If I have another child, the procedure may need to be
repeated.”
C. Use of cooling blanket
C. “This surgery may render me incapable of conceiving
D. Incentive spirometry another child.”

61. A client with a solar burn of the chest, back, face, and arms is seen D. “This procedure is accomplished in two separate
in urgent care. The nurse’s primary concern should be: surgeries.”

A. fluid resuscitation 66. A client experiences problems in body temperature regulation


associated with a skin impairment. Which gland is most likely
B. infection involved?

C. body image A. Eccrine

D. pain management B. Sebaceous

62. Which statement is true about crackles? C. Apocrine

A. They’re grating sounds. D. Endocrine

B. They’re high-pitched, musical squeaks. 67. A school cafeteria worker comes to the physician’s office
complaining of severe scalp itching. On inspection, the nurse finds nail
C. They’re low-pitched noises that sound like snoring. marks on the scalp and small light-colored round specks attached to
the hair shafts close to the scalp. These findings suggest that the client
D. They may be fine, medium, or course. suffers from:

63. A woman whose husband was recently diagnosed with active A. scabies
pulmonary tuberculosis (TB) is a tuberculin skin test converter.
Management of her care would include: B. head lice

A. scheduling her for annual tuberculin skin testing C. tinea capitis

B. placing her in quarantine until sputum cultures are D. impetigo


negative
68. Following a small-bowel resection, a client develops fever and
anemia. The surface surrounding the surgical wound is warm to touch
and necrotizing fasciitis is suspected. Another manifestation that B. Suggesting he take aspirin for relief because it’s
would most suggest necrotizing fasciitis is: probably early rheumatoid arthritis

A. erythema C. Validating his complaint but assuming it’s an adverse


effect of his vocation
B. leukocytosis
D. Asking him if he has been diagnosed or treated for
C. pressure-like pain carpal tunnel syndrome

D. swelling 74. The nurse is providing home care instructions to a client who has
recently had a skin graft. Which instruction is most important for the
69. A 28 yr-old nurse has complaints of itching and a rash of both client to remember?
hands. Contact dermatitis is initially suspected. The diagnosis is
confirmed if the rash appears: A. Use cosmetic camouflage techniques.

A. erythematous with raised papules B. Protect the graft from direct sunlight.

B. dry and scaly with flaking skin C. Continue physical therapy.

C. inflamed with weeping and crusting lesions D. Apply lubricating lotion to the graft site.

D. excoriated with multiple fissures 75. a 28 yr-old female nurse is seen in the employee health department
for mild itching and rash of both hands. Which of the following could
70. When assessing a client with partial thickness burns over 60% of be causing this reaction?
the body, which of the following should the nurse report immediately?
A. possible medication allergies
A. Complaints of intense thirst
B. current life stressors she may be experiencing
B. Moderate to severe pain
C. chemicals she may be using and use of latex gloves
C. Urine output of 70 ml the 1st hour
D. recent changes made in laundry detergent or bath
D. Hoarseness of the voice soap.

71. A client is admitted to the hospital following a burn injury to the 76. The nurse assesses a client with urticaria. The nurse understands
left hand and arm. The client’s burn is described as white and leathery that urticaria is another name for:
with no blisters. Which degree of severity is this burn?
A. hives
A. first-degree burn
B. a toxin
B. second-degree burn
C. a tubercle
C. third-degree burn
D. a virus
D. fourth-degree burn
77. A client with psoriasis visits the dermatology clinic. When
72. The nurse is caring for client with a new donor site that was inspecting the affected areas, the nurse expects to see which type of
harvested to treat a new burn. The nurse position the client to: secondary lesion?

A. allow ventilation of the site A. scale

B. make the site dependent B. crust

C. avoid pressure on the site C. ulcer

D. keep the site fully covered D. scar

73. a 45-yr-old auto mechanic comes to the physician’s office because 78. The nurse is caring for a bedridden, elderly adult. To prevent
an exacerbation of his psoriasis is making it difficult to work. He tells pressure ulcers, which intervention should the nurse include in the plan
the nurse that his finger joints are stiff and sore in the morning. The of care?
nurse should respond by:
A. Turn and reposition the client a minimum of every 8
A. Inquiring further about this problem because psoriatic hours.
arthritis can accompany psoriasis vulgaris
B. Vigorously massage lotion into bony prominences.
C. Post a turning schedule at the client’s bedside. A. A urine output consistently above 100 ml/hour.

D. Slide the client, rather than lifting when turning. B. A weight gain of 4 lb (1.8 kg) in 24 hours.

79. Following a full-thickeness (3rd degree) burn of his left arm, a C. Body temperature readings all within normal limits
client is treated with artificial skin. The client understands
postoperative care of the artificial skin when he states that during the D. An electrocardiogram (ECG) showing no
first 7 days after the procedure, he’ll restrict: arrhythmias.

A. range of motion 84. The nurse is reviewing the laboratory results of a client with
rheumatoid arthritis. Which of the following laboratory results should
B. protein intake the nurse expect to find?

C. going outdoors A. Increased platelet count

D. fluid ingestion B. Elevated erythrocyte sedimentation rate (ESR)

80. A client received burns to his entire back and left arm. Using the C. Electrolyte imbalance
Rule of Nines, the nurse can calculate that he has sustained burns on
what percentage of his body? D. Altered blood urea nitrogen (BUN) and creatinine
levels
A. 9%
85. Which nursing diagnosis takes the highest priority for a client with
B. 18% Parkinson’s crisis?

C. 27% A. Imbalanced nutrition: less than body requirements

D. 36% B. Ineffective airway clearance

81. The nurse is providing care for a client who has a sacral pressure C. Impaired urinary elimination
ulcer with wet-to-dry dressing. Which guideline is appropriate for a
wet-to-dry dressing? D. Risk for injury

A. The wound should remain moist form the dressing. 86. A client with a spinal cord injury and subsequent urine retention
receives intermittent catheterization every 4 hours. The average
B. The wet-to-dry dressing should be tightly packed into catheterized urine volume has been 550 ml. The nurse should plan to:
the wound.
A. Increase the frequency of the catheterizations.
C. The dressing should be allowed to dry out before
removal. B. Insert an indwelling urinary catheter

D. A plastic sheet-type dressing should cover the wet C. Place the client on fluid restrictions
dressing.
D. Use a condom catheter instead of an invasive one.
82. While in skilled nursing facility, a client contracted scabies, which
is diagnosed the day after discharge. The client is living at 87.The nurse is caring for a client who is to undergo a lumbar puncture
her daughter’s home with six other persons. During her visit to the to assess for the presence of blood in the cerebrospinal fluid (CSF) and
clinic, she asks a staff nurse, “What should my family do?” the most to measure CSF pressure. Which result would indicate n abnormality?
accurate response from the nurse is:
A. The presence of glucose in the CSF.
A. “All family members will need to be treated.”
B. A pressure of 70 to 200 mm H2O
B. “If someone develops symptoms, tell him to see a
physician right away.”
C. The presence of red blood cells (RBCs) in the first
specimen tube
C. “Just be careful not to share linens and towels with
family members.”
D. A pressure of 00 to 250 mmH2O
D. “After you’re treated, family members won’t be at risk
88. The nurse is administering eyedrops to a client with glaucoma. To
for contracting scabies.”
achieve maximum absorption, the nurse should instill the eyedrop into
the:
83. In an industrial accident, client who weighs 155 lb (70.3 kg)
sustained full-thickness burns over 40% of his body. He’s in the burn
A. conjunctival sac
unit receiving fluid resuscitation. Which observation shows that the
fluid resuscitation is benefiting the client?
B. pupil
C. sclera D. contrecoup

D. vitreous humor 94. When the nurse performs a neurologic assessment on Anne Jones,
her pupils are dilated and don’t respond to light.
89. A 52 yr-old married man with two adolescent children is beginning
rehabilitation following a cerebrovascular accident. As the nurse is A. glaucoma
planning the client’s care, the nurse should recognize that his condition
will affect: B. damage to the third cranial nerve

A. only himself C. damage to the lumbar spine

B. only his wife and children D. Bell’s palsy

C. him and his entire family 95. A 70 yr-old client with a diagnosis of leftsided cerebrovascular
accident is admitted to the facility. To prevent the development of
D. no one, if he has complete recovery diffuse osteoporosis, which of the following objectives is most
appropriate?
90. Which action should take the highest priority when caring for a
client with hemiparesis caused by a cerebrovascular accident (CVA)? A. Maintaining protein levels.

A. Perform passive range-of-motion (ROM) exercises. B. Maintaining vitamin levels.

B. Place the client on the affected side. C. Promoting weight-bearing exercises

C. Use hand rolls or pillows for support. D. Promoting range-of-motion (ROM) exercises

D. Apply antiembolism stockings 96. A client is admitted with a diagnosis of meningitis caused by
Neisseria meningitides. The nurse should institute which type of
91. The nurse is formulating a teaching plan for a client who has just isolation precautions?
experienced a transient ischemic attack (TIA). Which fact should the
nurse include in the teaching plan? A. Contact precautions

A. TIA symptoms may last 24 to 48 hours. B. Droplet precautions

B. Most clients have residual effects after having a TIA. C. Airborne precautions

C. TIA may be a warning that the client may have D. Standard precautions
cerebrovascular accident (CVA)
97. A young man was running along an ocean pier, tripped on an
D. The most common symptom of TIA is the inability to elevated area of the decking, and struck his head on the pier railing.
speak. According to his friends, “He was unconscious briefly and then
became alert and behaved as though nothing had happened.” Shortly
92. The nurse has just completed teaching about postoperative activity afterward, he began complaining of a headache and asked to be taken
to a client who is going to have a cataract surgery. The nurse knows to the emergency department. If the client’s intracranial pressure (ICP)
the teaching has been effective if the client: is increasing, the nurse would expect to observe which of the
following signs first?
A. coughs and deep breathes postoperatively
A. pupillary asymmetry
B. ties his own shoes
B. irregular breathing pattern
C. asks his wife to pick up his shirt from the floor after
he drops it. C. involuntary posturing

D. States that he doesn’t need to wear an eyepatch or D. declining level of consciousness


guard to bed
98. Emergency medical technicians transport a 28 yr-old iron worker
93. The least serious form of brain trauma, characterized by a brief loss to the emergency department. They tell the nurse, “He fell from a two-
of consciousness and period of confusion, is called: story building. He has a large contusion on his left chest and a
hematoma in the left parietal area. He has compound fracture of his
A. contusion left femur and he’s comatose. We intubated him and he’s maintaining
an arterial oxygen saturation of 92% by pulse oximeter with a manual-
resuscitation bag.” Which intervention by the nurse has the highest
B. concussion
priority?
C. coup
A. Assessing the left leg
B. Assessing the pupils 5. ANS: D
The portion of the cerebrum that controls speech and
C. Placing the client in Trendelenburg’s position hearing is the temporal lobe. Injury to the frontal lobe
causes personality changes, difficulty speaking, and
D. Assessing the level of consciousness disturbance in memory, reasoning, and concentration.
Injury to the parietal lobe causes sensory alterations
and problems with spatial relationships. Damage to
99. Alzheimer’s disease is the secondary diagnosis of a client admitted
the occipital lobe causes vision disturbances.
with myocardial infarction. Which nursing intervention should appear
on this client’s plan of care?
6. ANS: D
A. Perform activities of daily living for the client to Diabetes insipidus is an abrupt onset of extreme
polyuria that commonly occurs in clients after brain
decease frustration.
surgery. Cushing’s syndrome is excessive
glucocorticoid secretion resulting in sodium and water
B. Provide a stimulating environment. retention. Diabetes mellitus is a hyperglycemic state
marked by polyuria, polydipsia, and polyphagia.
C. Establish and maintain a routine. Adrenal crisis is undersecretion of glucocorticoids
resulting in profound hypoglycemia, hypovolemia,
D. Try to reason with the client as much as possible. and hypotension.

100. For a client with a head injury whose neck has been stabilized, 7. ANS: C
the preferred bed position is: The client should report the presence of foulsmelling
or cloudy urine. Unless contraindicated, the client
A. Trendelenburg’s should be instructed to drink large quantities of fluid
each day to flush the kidneys. Sand-like debris is
B. 30-degree head elevation normal because of residual stone products. Hematuria
is common after lithotripsy.
C. flat
8. ANS: A
D. side-lying A serum glucose level of 618 mg/dl indicates
hyperglycemia, which causes polyuria and deficient
fluid volume. In this client, tachycardia is more likely
Answers and Rationales
to result from deficient fluid volume than from
decreased cardiac output because his blood pressure is
1. ANS: D normal. Although the client’s serum glucose is
Because the client’s gag reflex is absent, elevating the elevated, food isn’t a priority because fluids and
head of the bed to 30 degrees helps minimize the insulin should be administered to lower the serum
client’s risk of aspiration. Checking the stools, glucose level. Therefore, a diagnosis of Imbalanced
performing ROM exercises, and keeping the skin Nutrition: Less then body requirements isn’t
clean and dry are important, but preventing aspiration appropriate. A temperature of 100.6º F isn’t life
through positioning is the priority. threatening, eliminating ineffective thermoregulation
as the top priority.
2. ANS: A
Any hole, no matter how small, will destroy the odor- 9. ANS: C
proof seal of a drainage bag. Removing the bag or Regular insulin, which is a short-acting insulin, has an
unclamping it is the only appropriate method for onset of 15 to 30 minutes and a peak of 2 to 4 hours.
relieving gas. Because the nurse gave the insulin at 2 p.m., the
expected onset would be from 2:15 to 2:30 p.m. and
3. ANS: A the peak from 4 p.m. to 6 p.m.
because celiac disease destroys the absorbing surface
of the intestine, fat isn’t absorbed but is passed in the 10. ANS: A
stool. Steatorrhea is bulky, fatty stools that have a foul CPP is derived by subtracting the ICP from the mean
odor. Jaundiced sclerae result from elevated bilirubin arterial pressure (MAP). For adequate cerebral
levels. Clay-colored stools are seen with biliary perfusion to take place, the minimum goal is 70
disease when bile flow is blocked. Celiac disease mmHg. The MAP is derived using the following
doesn’t cause a widened pulse pressure. formula:
MAP = ((diastolic blood pressure x 2) + systolic blood
4. ANS: D pressure) / 3
Reducing sodium intake reduces fluid retention. Fluid MAP = ((60 x2) + 90) / 3
retention increases blood volume, which changes MAP = 70 mmHg
blood vessel permeability and allows plasma to move To find the CPP, subtract the client’s ICP from the
into interstitial tissue, causing edema. Urea nitrogen MAP; in this case , 70 mmHg – 18 mmHg = 52
excretion can be increased only by improved renal mmHg.
function. Sodium intake doesn’t affect the glomerular
filtration rate. Potassium absorption is improved only 11. ANS: B
by increasing the glomerular filtration rate; it isn’t Breast cancer tumors are fixed, hard, and poorly
affected by sodium intake. delineated with irregular edges. Nipple retraction —
not eversion—may be a sign of cancer. A mobile mass
that is soft and easily delineated is most often a fluid- ammonia. Therefore, the client must limit his intake
filled benigned cyst. Axillary lymph nodes may or of sodium, meat (high in Protein), bananas (high in
may not be palpable on initial detection of a cancerous potassium), and fluid because the kidneys can’t
mass. secrete adequate urine.

12. ANS: D 20. ANS: D


An enterostomal nurse therapist is a registered nurse The client isn’t withdrawn or showing other signs of
who has received advance education in an accredited anxiety or depression. Therefore, the nurse can
program to care for clients with stomas. The probably safely approach her about talking with others
enterostomal nurse therapist can assist with selection who have had similar experiences, either through
of an appropriate stoma site, teach about stoma care, Reach for Recovery or another formal support group.
and provide emotional support. The nurse may educate the client’s spouse or partner
to listen to concerns, but the nurse shouldn’t tell the
13. ANS: A client’s spouse what to do. The client must consult
Ottorrhea and rhinorrhea are classic signs of basilar with her physician and make her own decisions
skull fracture. Injury to the dura commonly occurs about further treatment. The client needs to express
with this fracture, resulting in cerebrospinal fluid her sadness, frustration, and fear. She can’t be
(CSF) leaking through the ears and nose. Any fluid expected to be cheerful at all times.
suspected of being CSF should be checked for glucose
or have a halo test done. 21. ANS: C
HCG is one of the tumor markers for testicular cancer.
14. ANS: D The HCG level won’t identify the site of an infection
Testicular cancer commonly occurs in men between or evaluate prostatic function or testosterone level.
ages 20 and 30. A male client should be taught how to
perform testicular self-examination before age 20, 22. ANS: B
preferably when he enters his teens. Peripheral edema is a sign of fluid volume overload
and worsening heart failure. A skin rash, dry cough,
15. ANS: B and postural hypotension are adverse reactions to
Before weaning a client from mechanical ventilation, captopril, but the don’t indicate that therapy isn’t
it’s most important to have a baseline ABG levels. effective.
During the weaning process, ABG levels will be
checked to assess how the client is tolerating the 23. ANS: A
procedure. Other assessment parameters are less Tenting of chest skin when pinched indicates
critical. Measuring fluid volume intake and output is decreased skin elasticity due to dehydration. Hand
always important when a client is being mechanically veins fill slowly with dehydration, not rapidly. A pulse
ventilated. Prior attempts at weaning and ECG results that isn’t easily obliterated and neck vein distention
are documented on the client’s record, and the nurse indicate fluid overload, not dehydration.
can refer to them before the weaning process begins.
24. ANS: B
16. ANS: B The client should be encouraged to increase his
According to the ACS guidelines, “Women older than activity level. Maintaining an ideal weight; following
age 40 should perform breast selfexamination a low-cholesterol, low-sodium diet; and avoiding
monthly (not annually).” The hormonal receptor assay stress are all important factors in decreasing the risk
is done on a known breast tumor to determine whether of atherosclerosis.
the tumor is estrogen- or progesterone-dependent.
25. ANS: B
17. ANS: C Because thrombocytopenia impairs blood clotting, the
Increased pressure within the portal veins causes them nurse should assess the client regularly for signs of
to bulge, leading to rupture and bleeding into the bleeding, such as petechiae, purpura, epistaxis, and
lower esophagus. Bleeding associated with bleeding gums. The nurse should avoid administering
esophageal varices doesn’t stem from esophageal aspirin because it can increase the risk of bleeding.
perforation, pulmonary hypertension, or peptic ulcers. Frequent rest periods are indicated for clients with
anemia, not thrombocytopenia. Strict isolation is
18. ANS: B indicated only for clients who have highly contagious
Complex intra-articular fractures are repaired with or virulent infections that are spread by air or physical
external fixators because they have a better long-term contact.
outcome than those treated with casting. This is
especially true in a young client. The incidence of 26. ANS: B
complications, such as pin tract infections and SaO2 is the degree to which hemoglobin is saturated
neuritis, is 20% to 60%. Clients must be taught how to with oxygen. It doesn’t indicate the client’s overall
do pin care and assess for development of Hgb adequacy. Thus, an individual with a subnormal
neurovascular complications. Hgb level could have normal SaO2 and still be short
of breath. In this case, the nurse could assume that the
19. ANS: C client has a Hematologic problem. Poor peripheral
In a client with chronic renal failure, unrestricted perfusion would cause subnormal SaO2. There isn’t
intake of sodium, protein, potassium, and fluids may enough data to assume that the client’s problem is
lead to a dangerous accumulation of electrolytes and psychosomatic. If the problem were
protein metabolic products, such as amino acids and
left-sided heart failure, the client would exhibit insulin deficiency is an inappropriate nursing
pulmonary crackles. diagnosis for the client.

27. ANS: A 34. ANS: C


Addisonian crisis results in Hyperkalemia; therefore, When performing a nutritional assessment, one of the
administering potassium chloride is contraindicated. first things the nurse should do is to assess what the
Because the client will be hyponatremic, normal client typically eats. The client shouldn’t be permitted
saline solution is indicated. Hydrocortisone and to eat as desired. Weighing the client daily, placing
fludrocortisone are both useful in replacing deficient her on I & O status, and drawing blood to determine
adrenal cortex hormones. electrolyte level aren’t part of a nutritional
assessment.
28. ANS: D
Leukemia is manifested by an abnormal 35. Ans. C
overpopulation of immature leukocytes in the bone Regular follow-up care for the client with Grave’s
marrow. disease is critical because most cases eventually result
in hypothyroidism. Annual thyroid-stimulating
29. ANS: C hormone tests and the client’s ability to recognize
A sterile field is considered contaminated when it signs and symptoms of thyroid dysfunction will help
becomes wet. Moisture can act as a wick, allowing detect thyroid abnormalities early. Intake and output
microorganisms to contaminate the field. The outside is important for clients with fluid and electrolyte
of containers, such as sterile saline bottles, aren’t imbalances but not thyroid disorders. DDAVP is used
sterile. The containers should be opened before sterile to treat diabetes insipidus. While exercise to improve
gloves are put on and the solution poured over the cardiovascular fitness is important, for this client the
sterile dressings placed in a sterile basin. Wounds importance of regular follow-up is most critical.
should be cleaned from the most contaminated area to
the least contaminated area—for example, from the 36. ANS: D
center outward. The outer inch of a sterile field Anxiety related to the threat of death is an appropriate
shouldn’t be considered sterile. nursing diagnosis because the client’s anxiety can
adversely affect hear rate and rhythm by stimulating
30. ANS: C the autonomic nervous system. Also, because the
Caffeine is a stimulant, which can exacerbate client required resuscitation, the threat of death is a
palpitations and should be avoided by a client with real and immediate concern. Unless anxiety is dealt
symptomatic mitral valve prolapse. High fluid intake with first, the client’s emotional state will impede
helps maintain adequate preload and cardiac output. learning. Client teaching should be limited to clear
Aerobic exercise helps in increase cardiac output and concise explanations that reduce anxiety and promote
decrease heart rate. Protein-rich foods aren’t restricted cooperation. An anxious client has difficulty learning,
but high calorie foods are. so the deficient knowledge would continue despite
attempts teaching. Impaired physical mobility and
31. ANS: A social isolation are necessitated by the client’s critical
Excessive of aldosterone in the adrenal cortex is condition; therefore, they aren’t considered problems
responsible for the client’s hypertension. This warranting nursing diagnoses.
hormone acts on the renal tubule, where it promotes
reabsorption of sodium and excretion of potassium 37. ANS: A
and hydrogen ions. The pancreas mainly secretes Because tuberculosis is transmitted by droplet nuclei
hormones involved in fuel metabolism. The adrenal from the respiratory tract, the nurse should put on a
medulla secretes the cathecolamines—epinephrine mask when entering the client’s room. Having the
and norepinephrine. The parathyroids secrete client wear a mask at all the times would hinder
parathyroid hormone. sputum expectoration and make the mask moist from
respirations. If no contact with the client’s blood or
32. ANS: D body fluids is anticipated, the nurse need not wear a
A heart valve prosthesis, such as a mitral valve gown or gloves when providing direct care. A client
replacement, is a major risk factor for infective with tuberculosis should be in a room with laminar air
endocarditis. Other risk factors include a history of flow, and the door should be closed at all times.
heart disease (especially mitral valve prolapse),
chronic debilitating disease, IV drug abuse, and 38. ANS: C
immunosuppression. Although diabetes mellitus may The nurse can gently irrigate the tube but must take
predispose a person to cardiovascular disease, it isn’t care not to reposition it. Repositioning can cause
a major risk factor for infective endocarditis, nor is an bleeding. Suction should be applied continuously, not
appendectomy or pernicious anemia. every hour. The NG tube shouldn’t be clamped
postoperatively because secretions and gas will
33. ANS: A accumulate, stressing the suture line.
The client has signs and symptoms of diabetes
insipidus, probably caused by the failure of her renal 39. ANS: A
tubules to respond to antidiuretic hormone as a Regardless of the client’s medical history, rapid fluid
consequence of pyelonephritis. The hypernatremia is resuscitation is critical for maintaining cardiovascular
secondary to her water loss. Imbalanced nutrition integrity. Profound intravascular depletion requires
related to hypermetabolic state or catabolic effect of aggressive fluid replacement. A typical fluid
resuscitation protocol is 6 L of fluid over the first 12
hours, with more fluid to follow over the next 24 bearing joints such as the hips. This joint stiffness
hours. Various fluids can be used, depending on the alters functional ability and range of motion, placing
degree of hypovolemia. Commonly prescribed fluids the client at risk for falling and injury. Therefore,
include dextran (in case of hypovolemic shock), client safety is in jeopardy. Osteoporosis doesn’t
isotonic normal saline solution and, when the client is affect urinary elimination, breathing, or nutrition.
stabilized, hypotonic half-normal saline solution.
47. ANS: A
40. ANS: D PTH stimulates the kidneys to reabsorb calcium and
Glipizide may cause adverse skin reactions, such as excrete phosphate and converts vit D to its active
pruritus, and photosensitivity. It doesn’t cause form: 1 , 25 dihydroxy vitamin D. PTH doesn’t have
headache, constipation, or hypotension. a role in the metabolism of Vit E.

41. ANS: B 48. ANS: C


When clients are on mechanical ventilation, the The presence of unsecured area rugs poses a hazard in
artificial airway impairs the gag and cough reflexes all homes, particularly in one with a resident at high
that help keep organisms out of the lower respiratory risk for falls.
tract. The artificial airway also prevents the upper
respiratory system from humidifying and heating air 49. ANS: B
to enhance mucociliary clearance. Manipulations of Autonomy ascribes the right of the individual to make
the artificial airway sometimes allow secretions into his own decisions. In this case, the client is capable of
the lower airways. Whit standard procedures the other making his own decision and the nurse should support
choices wouldn’t be at high risk. his autonomy. Beneficence and justice aren’t the
principles that directly relate to the situation.
42. ANS: C Advocacy is the nurse’s role in supporting the
In chronic bronchitis, the diaphragmatic is flat and principle of autonomy.
weak. Diaphragmatic breathing helps to strengthen
the diaphragm and maximizes ventilation. Exhalation 50. ANS: D
should longer than inhalation to prevent collapse of Because respirations are depressed in myxedema
the bronchioles. The client with chronic bronchitis coma, maintaining a patent airway is the most critical
should exhale through pursed lips to prolong nursing intervention. Ventilatory support is usually
exhalation, keep the bronchioles from collapsing, and needed. Thyroid replacement will be administered IV.
prevent air trapping. Diaphragmatic breathing—not Although myxedema coma is associated with severe
chest breathing—increases lung expansion. hypothermia, a warming blanket shouldn’t be used
because it may cause vasodilation and shock. Gradual
43. ANS: A warming blankets would be appropriate. Intake and
Verbalizing the observed behavior is a therapeutic output are very important but aren’t critical
communication technique in which the nurse interventions at this time.
acknowledges what the client is feeling. Offering to
listen to the client express her anger can help the nurse 51. ANS: A
and the client understand its cause and begin to deal Glipizide begins to act in 15 to 30 minutes. The other
with it. Although stress can exacerbate the symptoms options are incorrect.
of SLE, telling the client to calm down doesn’t
acknowledge her feelings. Offering to get the nursing 52. ANS: A
supervisor also doesn’t acknowledge the client’s Hypoxia is the main breathing stimulus for a client
feelings. Ignoring the client’s feelings suggest that the with COPD. Excessive oxygen administration may
nurse has no interest in what the client has said. lead to apnea by removing that stimulus. Anginal pain
results from a reduced myocardial oxygen supply. A
44. ANS: A client with COPD may have anginal pain from
Daily walks relieve symptoms of intermittent generalized vasoconstriction secondary to hypoxia;
claudication, although the exact mechanism is however, administering oxygen at any concentration
unclear. Anaerobic exercise may exacerbate these dilates blood vessels, easing anginal pain. Respiratory
symptoms. Clients with chronic arterial occlusive alkalosis results from alveolar hyperventilation, not
disease must reduce daily fat intake to 30% or less of excessive oxygen administration. In a client with
total calories. The client should limit dietary COPD, high oxygen concentrations decrease the
cholesterol because hyperlipidemia is associated with ventilatory drive, leading to respiratory acidosis, not
atherosclerosis, a known cause of arterial occlusive alkalosis. High oxygen concentrations don’t cause
disease. However, HDLs have the lowest cholesterol metabolic acidosis.
concentration,
so this client should eat foods that raise HDL levels. 53. ANS: C
Continuous subcutaneous insulin regimen uses a basal
45. ANS: A rate and boluses of short-acting insulin. Multiple daily
Gastric decompression is typically low pressure and injection therapy uses a combination of short-acting
intermittent. High pressure and continuous gastric and intermediate or long-acting insulins.
suctioning predisposes the gastric mucosa to injury
and ulceration. 54. ANS: B
Holding a cane on the uninvolved side distributes
46. ANS: A weight away from the involved side. Holding the cane
In osteoarthritis, stiffness is common in large, weight
close to the body prevents leaning. Use of a cane a concern that has a lower priority than pain
won’t maintain stride length or prevent edema. management.

55. ANS: D 62. ANS: D


High pulmonary artery wedge pressures are diagnostic Crackles result from air moving through airways that
for left-sided heart failure. With leftsided heart failure, contain fluid. Heard during inspiration and expiration,
pulmonary edema can develop causing pulmonary crackles are discrete sounds that vary in pitch and
crackles. In leftsided heart failure, hypotension may intensity. They’re classified as fine, medium, or
result and urine output will decline. Dry mucous coarse. Pleural friction rubs have a distinctive grating
membranes aren’t directly associated with elevated sound. As the name indicates, these breath sounds
pulmonary artery wedge pressures. result when inflamed pleurae rub together.
Continuous, highpitched, musical squeaks, called
56. ANS: B wheezes, result when air moves rapidly through
It’s mandatory in most settings to have a physician’s airways narrowed by asthma or infection or when an
order before restraining a client. A client should never airway is partially obstructed by a tumor or foreign
be left alone while the nurse summons assistance. All body. Wheezes, like gurgles, occur on expiration and
staff members require annual instruction on the use of sometimes on inspiration. Loud, coarse, low-pitched
restraints, and the nurse should be familiar with the sounds resembling snoring are called gurgles. These
facility’s policy. sounds develop when thick secretions partially
obstruct airflow through the large upper airways.
57. ANS: A
The client who has undergone a thyroidectomy is t risk 63. Ans. D
for developing hypocalcemia from inadvertent Individuals who are tuberculin skin test converters
removal or damage to the parathyroid gland. The should begin a 6-month regimen of an antitubercular
client with hypocalcemia will exhibit a positive drug such as INH, and they should never have another
Chvostek’s sign (facial muscle contraction when the skin test. After an individual has a positive tuberculin
facial nerve in front of the ear is tapped) and a positive skin test, subsequent skin tests will cause severe skin
Trousseau’s sign (carpal spasm when a blood pressure reactions but won’t provide new information about the
cuff is inflated for few minutes). These signs aren’t client’s TB status. The client doesn’t have active TB,
present with hypercalcemia, hypokalemia, or so can’t transmit, or spread, the bacteria. Therefore,
Hyperkalemia. she shouldn’t be quarantined or asked for information
about recent contacts.
58. ANS: C
Diarrhea causes a bicarbonate deficit. With loss of the 64. ANS: C
relative alkalinity of the lower GI tract, the relative Disturbed body image is a negative perception of the
acidity of the upper GI tract predominates leading to self that makes healthful functioning more difficult.
metabolic acidosis. Diarrhea doesn’t lead to The defining characteristics for this nursing diagnosis
respiratory acid-base imbalances, such as respiratory include undergoing a change in body structure or
acidosis and respiratory alkalosis. Loss of acid, which function, hiding or overexposing a body part, not
occurs with severe vomiting, may lead to metabolic looking at a body part, and responding verbally or
alkalosis. nonverbally to the actual or perceived change in
structure or function. This client may have any of the
59. ANS: B other diagnoses, but the signs and symptoms
The head of the bed must be elevated while the client described in he case most closely match the defining
is eating. The client should be placed in a recumbent characteristics for disturbed body image.
position—not a supine position— when lying down to
reduce the risk of aspiration. Encourage the client to 65. ANS: B
wear properly fitted dentures to enhance his chewing Because the pregnant uterus exerts a lot of pressure on
ability. Thickened liquids, not thin liquids, decrease the urinary bladder, the bladder repair may need to be
aspiration risk. repeated. These clients don’t necessarily have to have
a cesarean delivery if they become pregnant, and this
60. ANS: A procedure doesn’t render them sterile. This procedure
Endotracheal suctioning secretions as well as gases is completed in one surgery.
from the airway and lowers the arterial oxygen
saturation (SaO2) level. Coughing and incentive 66. ANS: A
spirometry improve oxygenation and should raise or Eccrine glands are associated with body temperature
maintain oxygen saturation. Because of superficial regulation. Sebaceous glands lubricate the skin and
vasoconstriction, using a cooling blanket can lower hairs, and apocrine glands are involved in bacteria
peripheral oxygen saturation readings, but SaO2 decomposition. Endocrine glands secrete hormones
levels wouldn’t be affected. responsible for the regulation of body processes, such
as metabolism and glucose regulation.
61. ANS: D
With a superficial partial thickness burn such as a 67. ANS: B
solar burn (sunburn), the nurse’s main concern is pain The light-colored spots attached to the hair shafts are
management. Fluid resuscitation and infection nits, which are the eggs of head lice. They can’t be
become concerns if the burn extends to the dermal and brushed off the hair shaft like dandruff. Scabies is a
subcutaneous skin layers. Body image disturbance is contagious dermatitis caused by the itch mite,
Sacoptes scabiei, which lives just beneath the skin.
Tinea capitis, or ringworm, causes patchy hair loss protect the graft from sunlight. The other three
and circular lesions with healing centers. Impetigo is interventions are all helpful to the client and his
an infection caused by Staphylococcus or recovery but are less important.
Sterptococcus, manifested by vesicles or pustules that
form a thick, honey-colored crust. 75. ANS: C
Because the itching and rash are localized, an
68. ANS: C environmental cause in the workplace should be
Severe pressure-like pain out of proportion to visible suspected. With the advent of universal precautions,
signs distinguishes necrotizing fasciitis from many nurses are experiencing allergies to latex gloves.
cellulites. Erythema, leukocytosis, and swelling are Allergies to medications, laundry detergents, or bath
present in both cellulites and necrotizing fasciitis. soaps or a dermatologic reaction to stress usually elicit
a more generalized or widespread rash.
69. ANS: A
Contact dermatitis is caused by exposure to a physical 76. ANS: A
or chemical allergen, such as cleaning products, skin Hives and urticaria are two names for the same skin
care products, and latex gloves. Initial symptoms of lesion. Toxin is a poison. A tubercle is a tiny round
itching, erythema, and raised papules occur at the site nodule produced by the tuberculosis bacillus. A virus
of the exposure and can begin within 1 hour of is an infectious parasite.
exposure. Allergic reactions tend to be red and not
scaly or flaky. Weeping, crusting lesions are also 77. ANS: A
uncommon unless the reaction is quite severe or has A scale is the characteristic secondary lesion
been present for a long time. Excoriation is more occurring in psoriasis. Although crusts, ulcers, and
common in skin disorders associated with a moist scars also are secondary lesions in skin disorders, they
environment. don’t accompany psoriasis.

70. ANS: D 78. ANS: C


Hoarseness indicate injury to the respiratory system A turning schedule with a signing sheet will help
and could indicate the need for immediate intubation. ensure that the client gets turned and thus, help
Thirst following burns is expected because of the prevent pressure ulcers. Turning should occur every
massive fluid shifts and resultant loss leading to 1-2 hours—not every 8 hours—for clients who are in
dehydration. Pain, either severe or moderate, is bed for prolonged periods. The nurse should apply
expected with a burn injury. The client’s output is lotion to keep the skin moist but should avoid vigorous
adequate. massage, which could damage capillaries. When
moving the client, the nurse should lift rather than
71. ANS: C slide the client to void shearing.
Third-degree burn may appear white, red, or black and
are dry and leathery with no blisters. There may be 79. ANS: A
little pain because nerve endings have been destroyed. To prevent disruption of the artificial skin’s adherence
First-degree burns are superficial and involve the to the wound bed, the client should restrict range of
epidermis only. There is local pain and redness but no motion of the involved limb. Protein intake and fluid
blistering. Second-degree burn appear red and moist intake are important for healing and regeneration and
with blister formation and are painful. Fourth-degree shouldn’t be restricted. Going outdoors is acceptable
burns involve underlying muscle and bone tissue. as long as the left arm is protected from direct
sunlight.
72. ANS: C
A universal concern I the care of donor sites for burn 80. ANS: C
care is to keep the site away from sources of pressure. According to the Rule of Nines, the posterior and
Ventilation of the site and keeping the site fully anterior trunk, and legs each make up 18% of the total
covered are practices in some institutions but aren’t body surface. The head, neck, and arms each make up
hallmarks of donor site care. Placing the site in a 9% of total body durface, and the perineum makes up
position of dependence isn’t a justified aspect of donor 1%. In this case, the client received burns to his back
site care. (18%) and one arm (9%), totaling 27%.

73. ANS: A 81. ANS: A


Anyone with psoriasis vulgaris who reports joint pain A wet-to-dry saline dressing should always keep the
should be evaluated for psoriaic arthritis. wound moist. Tight packing or dry packing can cause
Approximately 15% to 20% of individuals with tissue damage and pain. A dry gauze —not a plastic-
psoriasis will also develop psoriatic arthritis, which sheet-type dressing—should cover the wet dressing.
can be painful and cause deformity. It would be
incorrect to assume that his pain is caused by early 82. ANS: A
rheumatoid arthritis or his vocation without asking When someone in a group of persons sharing a home
more questions or performing diagnostic studies. contracts scabies, each individual in the same home
Carpal tunnel syndrome causes sensory and motor needs prompt treatment whether he’s symptomatic or
changes in the fingers rather than localized pain in the not. Towels and linens should be washed in hot water.
joints. Scabies can be transmitted from one person to another
before symptoms develop
74. ANS: B
To avoid burning and sloughing, the client must
83. ANS: A family members and affects the entire family, even if
In a client with burns, the goal of fluid resuscitation is the client eventually recovers completely.
to maintain a mean arterial blood pressure that
provides adequate perfusion of vital structures. If the 90. ANS: B
kidneys are adequately perfused, they will produce an To help prevent airway obstruction and reduce the risk
acceptable urine output of at least 0.5 ml/kg/hour. of aspiration, the nurse should position a client with
Thus, the expected urine output of a 155-lb client is 35 hemiparesis on the affected side. Although
ml/hour, and a urine output consistently above 100 performing ROM exercises, providing pillows for
ml/hour is more than adequate. Weight gain from fluid support, and applying antiembolism stockings can be
resuscitation isn’t a goal. In fact, a 4 lb weight gain in appropriate for a client with CVA, the first concern is
24 hours suggests third spacing. Body temperature to maintain a patent airway.
readings and ECG interpretations may demonstrate
secondary benefits of fluid resuscitation but aren’t 91. ANS: C
primary indicators. TIA may be a warning that the client will experience
a CVA, or stroke, in the near future. TIA aymptoms
84. ANS: B last no longer than 24 hours and clients usually have
The ESR test is performed to detect inflammatory complete recovery after TIA. The most common
processes in the body. It’s a nonspecific test, so the symptom of TIA is sudden, painless loss of vision
health care professional must view results in lasting up to 24 hours.
conjunction with physical signs and symptoms.
Platelet count, electrolytes, BUN, and creatinine 92. ANS: C
levels aren’t usually affected by the inflammatory Bending to pick up something from the floor would
process. increase intraocular pressure, as would bending to tie
his shoes. The client needs to wear eye protection to
85. ANS: B bed to prevent accidental injury during sleep.
In Parkinson’s crisis, dopamine-related symptoms are
severely exacerbated, virtually immobilizing the 93. ANS: B
client. A client who is confined to bed during a crisis Concussions are considered minor with no structural
is at risk for aspiration and pneumonia. Also, signs of injury. A contusion is bruising of the brain
excessive drooling increases the risk of airway tissue with small hemorrhages in the tissue. Coup and
obstruction. Because of these concerns, ineffective contrecoup are type of injuries in which the damaged
airway clearance is the priority diagnosis for this area on the brain forms directly below that site of
client. Although imbalanced nutrition:less than body impact (coup) or at the
requirements, impaired urinary elimination and risk site opposite the injury (contrecoup) due to movement
for injury also are appropriate diagnoses for this client, of the brain within the skull.
they aren’t immediately lifethreatening and thus are
less urgent.
94. ANS: B
The third cranial nerve (oculomotor) is responsible for
86. ANS: A pupil constriction. When there is damage to the nerve,
As a rule of practice, if intermittent catheterization for the pupils remain dilated and don’t respond to light.
urine retention typically yields 500 ml or more, the Glaucoma, lumbar spine injury, and Bell’s palsy
frequency of catheterization should be increased. won’t affect pupil constriction.
Indwelling catheterization is less preferred because of
the risk of urinary tract infection and the loss of
95. ANS: C
bladder tone. Fluid restrictions aren’t indicated for this
When the mechanical stressors of weight bearing are
case; the problem isn’t overhydration, rather it’s urine
absent, diffuse osteoporosis can occur. Therefore, if
retention. A condom catheter doesn’t help empty the
the client does weight-bearing exercises, disuse
bladder of a client with urine retention.
complications can be prevented. Maintaining protein
and vitamins levels is important, but neither will
87. ANS: D prevent osteoporosis. ROM exercises will help
The normal pressure is 70 to 200 mm H2O are prevent muscle atrophy and contractures.
considered abnormal. The presence of glucose is an
expected finding in CSF, and RBCs typically occur in
96. ANS: B
the first specimen tube from the trauma caused by the
This client requires droplet precautions because the
procedure.
organism can be transmitted through airborne droplets
when the client coughs, sneezes, or doesn’t cover his
88. ANS: A mouth. Airborne precautions would be instituted for a
The nurse should instill the eyedrop into the client infected with tuberculosis. Standard precautions
conjunctival sac where absorption can best take place. would be instituted for a client when contact with
The pupil permits light to enter the eye. The sclera body substances is likely. Contact precautions would
maintains the eye’s shape and size. The vitreous be instituted for a client infected with an organism that
humor maintains the retina’s placement and the shape is transmitted through skin-to-skin
of the eye. contact.

89. ANS: C 97. ANS: D


According to family theory, any change in a family With a brain injury such as an epidural hematoma (a
member, such as illness, produces role changes in all diagnosis that is most likely based on this client’s
symptoms), the initial sign of increasing ICP is a
change in the level of consciousness. As neurologic A. necrosis
deterioration progresses, manifestations involving
pupillary symmetry, breathing patterns, and posturing B. injury
will
occur. C. ischemia

98. ANS: A D. nothing significant


In the scenario, airway and breathing are established
so the nurse’s next priority should be circulation. With
4. Red has just returned from the postanesthesia care unit (PACU) from
a compound fracture of the femur, there is a high risk
a hemorrhidectomy. His postoperative orders include sitz baths every
of profuse bleeding; therefore, the nurse should assess
morning. The nurse understands that sitz bath is use for:
the site. Neurologic assessment is a secondary concern
to airway, breathing and circulation. The nurse
doesn’t have enough data to warrant putting the client A. promote healing
in Trendelenburg’s position.
B. relive tension
99. ANS: C
Establishing and maintaining a routine is essential to C. lower body temperature
decreasing extraneous stimuli. The client should
participate in daily care as much as possible. D. cause swelling
Attempting to reason with such clients isn’t
successful, because they can’t participate in abstract 5. Trousseau’s sign is associated with which electrolyte imbalance?
thinking.
A. hyponatremia
100. ANS: B
For clients with increased intracranial pressure (ICP), B. hypocalcemia
the head of the bed is elevated to promote venous
outflow. Trendelenburg’s position is contraindicated C. hypernatremia
because it can raise ICP. Flat or neutral positioning is
indicated when elevating the head of the bed would
D. hypercalcemia
increase the risk of neck injury or airway obstruction.
Sidelying isn’t specifically a therapeutic treatment for
increased ICP. 6. A 36 year old female complains of headache and neck pain. The
nurse’s assessments reveal painful flexion of the neck to the chest. The
nurse understands that nuchal rigidity is associated with:

A. brain tumor
Medical Surgical Nursing Exam 3
B. CVA
1. Lisa is newly diagnosed with asthma and is being discharged from
the hospital after an episode of status asthmaticus. Discharge teaching C. meningitis
should include which of the following:
D. subdural hematoma
A. Limitations in sports that will be imposed by the
illness 7. The nurse teaching the client about behavioral changes, which can
affect development of atherosclerosis, should discuss which of the
B. Specific instructions on staying cal during an attack following as a non-modifiable risk factor for atherosclerosis?

C. The relationship of symptoms and a specific trigger A. cigarette smoking


such as physical exercise
B. hyperlipidemia
D. Incidence of status asthmaticus in children and teens
C. female over 55 years of age
2. Which of the following symptoms is most characteristic of a client
with a cancer of the lung? D. sedentary lifestyle

A. air hunger 8. A 76 year old man enters the ER with complaints of back pain and
feeling fatigued. Upon examination, his blood pressure is 190/100,
B. exertional dyspnea pulse is 118, and hematocrit and hemoglobin are both low. The nurse
palpates the abdomen which is soft, non-tender and auscultates an
C. cough with night sweats abdominal pulse. The most likely diagnosis is:

D. persistent changing cough A. Buerger’s disease

3. The client has ST segment depression on his 12-lead ECG. The B. CHF
nurse determines that this would indicate the following:
C. Secondary hypertension
D. Aneurysm 14. A shuffling gait is typically associated with the patient who has:

9. Nurse Fiona is caring a patient with Raynaud’s disease. Which of A. Parkinson’s disease
the following outcomes concerning medication regimen is of highest
priority? B. Multiple sclerosis

A. Controlling the pain once vasospasm occur C. Raynaud’s disease

B. Relaxing smooth muscle to avoid vasospasms D. Myasthenia gravis

C. Preventing major disabilities that may occur 15. The priority in preparing the room for a client with a C7 spinal cord
injury is having:
D. Avoiding lesions on the feet
A. the halo brace device
10. Mr. Roberto Robles complains of a severe headache and is
extremely anxious. The nurse checks his vital signs and finds him to B. a catheterization tray
have a heart rate of 57 bpm and a blood pressure of 230/110 mmHg.
The nurse should also assess for? C. a ventilator on stand by

A. presence of bowel sounds D. the spinal kinetic bed

B. presence of babinski reflex 16. A 47 year old man with liver failure who has developed ascites.
The nurse understands that ascites is due to:
C. fecal incontinence
A. dehydration
D. urinary catheter patency
B. protein deficiency
11. A 40n year old male patient is complaining of chronic progressive
and mental deterioration is admitted to the unit. The nurse recognizes C. bleeding disorders
that these characteristics indicate a disease that results in degeneration
of the basal ganglia and cerebral cortex. The disease is called:
D. vitamin deficiency
A. multiple sclerosis
17. A client with rheumatoid arthritis may reveal which of the
following assessment data:
B. myasthenia gravis
A. Heberden’s nodes
C. Huntington’s disease
B. Morning stiffness no longer than 30 minutes
D. Guillain-Barre syndrome
C. Asymmetric joint swelling
12. Dianne Hizon is a 27 year old woman who has been admitted to
the ER due to severe vomiting. Her ABG values are pH= 7.50,
D. Swan neck deformities
PaCO2= 85, HCO3= 31, and SaO2= 93%. The nurse interpretation of
this ABG analysis is:
18. Elsa Santos is a 18 year old student admitted to the ward with a
diagnosis of epilepsy. She tells the nurse that she is experiencing a
A. respiratory acidosis
generalized tingling sensation and is “smelling roses”. The nurse
understands that Esla is probably experiencing:
B. respiratory alkalosis
A. an acute alcohol withdrawal
C. metabolic acidosis
B. an acute CVA
D. metabolic alkalosis
C. an aura
13. Mr. Perkson has a parkinson’s disease and he finds the resting
tremor he is experiencing in his right hand very frustrating. The nurse
D. an olfactory hallucination
advises him to:
19. Mr. Lucas, a 63 year old, went to the clinic complaining of
A. take a warm bath
hoarseness of voice and a cough. His wife states that his voice has
changed in the last few months. The nurse interprets that Mr. Lucas’s
B. hold an object symptoms are consistent with which of the following disorders:

C. practice deep breathing A. chronic sinusitis

D. take diazepam as needed B. laryngeal cancer


C. gastroesophageal reflux disease D. Use of a clean razor blade each time he shaves

D. coronary artery disease 25. A 48 year old woman presents to the hospital complaining of chest
pain, tachycardia and dyspnea. On exam, heart sounds are muffled.
20. Sarah complains of a nursing sensation, cramping pain in the top Which of the following assessment findings would support a diagnosis
part of her abdomen that becomes worse in the afternoon and of cardiac tamponade?
sometimes awakes her at night. She reports that when she eats, it helps
the pain go away but that pain is now becoming more intense. Which A. A deviated trachea
of the following is the best condition for the nurse to draw:
B. Absent breath sounds to the lower lobes
A. these symptoms are consistent with an ulcer
C. Pulse 40 with inspiration
B. The client probably has indigestion
D. Blood pressure 140/80
C. A snack before going to bed should be advised
Answers and Rationales
D. The client probably developing cholelithiasis
1. C. The relationship of symptoms and a specific
21. Nurse Cynthia is providing a discharge teaching to a client with trigger such as physical exercise. COPD clients
chronic cirrhosis. His wife asks her to explain why there is so much have low oxygen and high carbon dioxide levels.
emphasis on bleeding precautions. Which of the following provides Therefore, hypoxia is the main stimulus for ventilation
the most appropriate response? is persons with chronic hypercapnea. Increasing the
level of oxygen would decrease the stimulus to
A. “The low protein diet will result in reduced clotting.” breathe.

B. “The increased production of bile decreases clotting 2. D. persistent changing cough. The most common
factors.” sign of lung cancer is a persistent cough that changes.
Other signs are dyspnea, bloody sputum and long term
C. “The liver affected by cirrhosis is unable to produce pulmonary infection. Option A is common with
clotting factors.” asthma, option B is common with COPD and option
C is common with TB.
D. “The required medications reduce clotting factors.”
3. C. ischemia. Depressed ST segment and inverted T-
22. Betty Lee is a 58 year old woman who is being admitted to the waves represent myocardial ischemia. Injury has a ST
medical ward with trigeminal neuralgia. The nurse anticipates that Mr. segment elevation.
Lee will demonstrate which of the following major complaints?
4. A. promote healing. Sitz bath provides moist heat to
A. excruciating, intermittent, paroxysmal facial pain the perineal and anal area to clean, promote healing
and drainage and reduce soreness to the area. Sitz bath
helps healing with cleaning action and promotion of
B. unilateral facial droop
circulation, thereby reducing swelling. Sitz bath
usually has no therapeutic value in lowering body
C. painless eye spasm temperature. Although relief of tension can occur, this
effect is secondary to the promotion of healing.
D. mildly painful unilateral eye twitching
5. B. hypocalcemia. Trousseau’s sign is a carpal pedal
23. A 38 year old woman returns from a subtotal thryroidectomy for spasm elicited when a blood pressure cuff is inflated
the treatment of hyperthyroidism. Upon assessment, the immediate on the arm of a patient with hypocalcemia.
priority that the nurse would include is:
6. C. meningitis. A patient with meningitis will exhibit
A. Assess for pain signs that include photophobia and nuchal rigidity,
which is pain on the flexion of the chin to chest.
B. Assess for neurological status
7. C. female over 55 years of age. Lifestyle, cigarette
C. Assess fluid volume status smoking and hyperlipidemia can be changed by
changing behaviors.
D. Assess for respiratory distress
8. D. Aneurysm. The symptoms exhibited by the client
24. Nurse Shiela is teaching self-care to a client with psoriasis. The are typical of an abdominal aortic aneurysm. The most
nurse should encourage which of the following for his scaled lesion? significant sign is the audible pulse in the abdominal
area. If hemorrhage were present, the abdomen would
A. Importance of follow-up appointments be tender and firm.

B. Emollients and moisturizers to soften scales 9. B. Relaxing smooth muscle to avoid


vasospasms. The major task of the health care team
C. Keep occlusive dressings on the lesions 24 hours a day is to medicate the client drugs that produce smooth
muscle relaxation, which will decrease the vasospasm 19. B. laryngeal cancer. These symptoms, along with
and increase the arterial flow to the affected part. The dysphagia, foul-smelling breath, and pain when
drugs used are calcium antagonists. drinking hot or acidic, are common signs of laryngeal
cancer.
10. D. urinary catheter patency. The patient is
complaining of symptoms of autonomic dysreflexia, 20. A. these symptoms are consistent with an
which consists of the triad of hypertension, ulcer. The description of pain is consistent with ulcer
bradycardia and a headache. Major causes of pain. The pain is epigastric and is worse when the
autonomic dysreflexia include urinary bladder stomach is empty and is relived by food.
distention and fecal impaction. Checking the patency
of the urinary catheter will check for bladder 21. C. “The liver affected by cirrhosis is unable to
distention. produce clotting factors.” When bile production is
reduced, the body has reduced ability to absorb fat-
11. C. Huntington’s disease. Huntington’s disease is a soluble vitamins. Without adequate Vitamin K
hereditary disease in which degeneration of the basal absorption, clotting factors II, VII, IX, and X are not
ganglia and cerebral cortex causes chronic progressive produced in sufficient amounts.
chorea (muscle twitching) and mental deterioration,
ending in dementia. Huntington’s disease usually 22. A. excruciating, intermittent, paroxysmal facial
strikes people ages 25 to 55. pain. Trigeminal neuralgia is a syndrome of
excruciating, intermittent, paroxysmal facial pain. It
12. D. metabolic alkalosis. Ms. Hizon’s pH is above manifests as intense, periodic pain in the lips, gums,
7.45, which makes it alkalatic, and her bicarbonate is teeth or chin. The other symptoms aren’t characteristic
high which is also makes it basic. Thus, the diagnosis of trigeminal neuralgia.
is metabolic alkalosis.
23. D. Assess for respiratory distress. Though fluid
13. B. hold an object. The resting or non-intentional volume status, neurological status and pain are all
tremor may be controlled with purposeful movement important assessment, the immediate priority for
such as holding an object. A warm bath, deep postoperative is the airway management. Respiratory
breathing and diazepam will promote relaxation but distress may result from hemorrhage, edema,
are not specific interventions for tremor. laryngeal damage or tetany.

14. A. Parkinson’s disease. A shuffling gait from the 24. B. Emollients and moisturizers to soften
musculoskeletal rigidity of the patient with scales. Emollients will ease dry skin that increases
Parkinson’s disease is common. Patients experiencing pruritus and causes psoriasis to be worse. Washing
a stroke usually exhibit loss of voluntary control over and drying the skin with rough linens or pressure may
motor movements associated with generalized cause excoriation. Constant occlusion may increase
weakness; a shuffling gait is usually not observed in the effects of the medication and increase the risk of
stroke patient. infection.

15. C. a ventilator on stand by. Although a ventilator is 25. C. Pulse 40 with inspiration. Paradoxical pulse is a
not required for injury below C3, the innervation of hallmark symptom of cardiac tamponade. As pressure
intercostal muscles is affected. Hemorrhage and cord is exerted on the left ventricle from fluid, the natural
swelling extends the level of injury making it likely increase in pressure from the right ventricle during
that this client will need a ventilator. inspiration creates even more pressure, diminishing
cardiac output.
16. B. protein deficiency. Protein deficiency allows fluid
to leak out of the vascular system and third space into
the tissues and spaces in the body such as the
peritoneal space. Bleeding tendencies, dehydration
and vitamin deficiency can occur but don’t cause Medical Surgical Nursing Exam 4
ascites.
1. A patient is admitted to the medical surgical unit following surgery.
17. D. Swan neck deformities. Swan neck deformities of Four days after surgery, the patient spikes a 38.9 degrees C oral
the hand are classic deformities associated with temperature and exhibits a wet, productive cough. The nurse assesses
rheumatoid arthritis secondary to the presence of the patient with understanding that an infection that is acquired during
fibrous connective tissue within the joint space. hospitalization is known as:
Clients with RA do experience morning stiffness, but
it can last from 30 minutes up to several hours. RA is A. a community acquired infection
characterized by symmetrical joint movement, and
heberden’s nodes are characteristic of osteoarthritis. B. an iatrogenic infection

18. C. an aura. An aura frequently precedes an epileptic C. a nosocomial infection


seizure and may manifest as vague physic discomfort
or specific aromas. Patients experiencing auras aren’t D. an opportunistic infection
having a CVA, substance withdrawal or hallucination.
2. A client with anemia has a hemoglobin of 6.5 g/dL. The client is
experiencing symptoms of cerebral tissue hypoxia. Which of the
following nursing interventions would be most important in providing A. Apply heat compress to the affected area
care?
B. Immobilize the affected area
A. Providing rest periods throughout the day
C. Administer narcotic analgesics for pain
B. Instituting energy conservation techniques
D. Administer OTC analgesics for pain
C. Assisting in ambulation to the bathroom
8. A client with congestive heart failure has digoxin (Lanoxin) ordered
D. Checking temperature of water prior to bathing everyday. Prior to giving the medication, the nurse checks the digoxin
level which is therapeutic and ausculates an apical pulse. The apical
3. A client was involved in a motor vehicular accident in which the pulse is 63 bpm for 1 full minute. The nurse should:
seat belt was not worn. The client is exhibiting crepitus, decrease
breath sounds on the left, complains of shortness of breath, and has a A. Hold the Lanoxin
respiratory rate of 34 breaths per minute. Which of the following
assessment findings would concern the nurse most? B. Give the half dose now, wait an hour and give the
other half
A. Temperature of 102 degrees F and productive cough
C. Call the physician
B. ABG with PaO2 of 92 and PaCO2 of 40 mmHg
D. Give the Lanoxin as ordered
C. Trachea deviating to the right
9. Nurse Marian is caring for a client with haital hernia, which of the
D. Barrel-chested appearance following should be included in her teaching plan regarding causes:

4. The proper way to open an envelop-wrapped sterile package after A. To avoid heavy lifting
removing the outer package or tape is to open the first position of the
wrapper: B. A dietary plan based on soft foods

A. away from the body C. Its prevalence in young adults

B. to the left of the body D. Its prevalence in fair-skinned individuals

C. to the right of the body 10. Joseph has been diagnosed with hepatic encephalopathy. The nurse
observes flapping tremors. The nurse understands that flapping
D. toward the body tremors associated with hepatic encephalopathy are also known as:

5. Assessment of a client with possible thrombophlebitis to the left leg A. aphasia


and a deep vein thrombosis is done by pulling up on the toes while
gently holding down on the knee. The client complains of extreme pain B. ascites
in the calf. This should be documented as:
C. astacia
A. positive tourniquet test
D. asterixis
B. positive homan’s sign
11. Hyperkalemia can be treated with administration of 50% dextrose
C. negative homan’s sign and insulin. The 50% dextrose:

D. negative tourniquet test A. causes potassium to be excreted

6. Thomas Elison is a 79 year old man who is admitted with diagnosis B. causes potassium to move into the cell
of dementia. The doctor orders a series of laboratory tests to determine
whether Mr. Elison’s dementia is treatable. The nurse understands that C. causes potassium to move into the serum
the most common cause of dementia in this population is:
D. counteracts the effects of insulin
A. AIDS
12. Which of the following findings would strongly indicate the
B. Alzheimer’s disease possibility of cirrhosis?

C. Brain tumors A. dry skin

D. Vascular disease B. hepatomegaly

7. Which of the following nursing interventions is contraindicated in C. peripheral edema


the care of a client with acute osteomyelitis?
D. pruritus B. Monitor temperature every 4 hours

13. Aling Puring has just been diagnosed with close-angle (narrow- C. Instruct him to avoid sexual contact during acute
angle) glaucoma. The nurse assesses the client for which of the phases of illness
following common presenting symptoms of the disorder?
D. Encourage him to use antifungal agents regularly
A. halo vision
19. An 8 year old boy is brought to the trauma unit with a chemical
B. dull eye pain burn to the face. Priority assessment would include which of the
following?
C. severe eye and face pain
A. Skin integrity
D. impaired night vision
B. BP and pulse
14. Chvostek’s sign is associated with which electrolyte impabalnce?
C. Patency of airway
A. hypoclacemia
D. Amount of pain
B. hypokalemia
20. A client with anemia due to chemotherapy has a hemoglobin of 7.0
C. hyponatremia g/dL. Which of the following complaints would be indicative of tissue
hypoxia related to anemia?
D. hypophosphatenia
A. dizziness
15. What laboratory test is a common measure of the renal function?
B. fatigue relieved by rest
A. CBC
C. skin that is warm and dry to the touch
B. BUN/Crea
D. apathy
C. Glucose
21. Hazel Murray, 32 years old complains of abrupt onset of chest and
D. Alanine amino transferase (ALT) back pain and loss of radial pulses. The nurse suspects that Mrs.
Murray may have:
16. Nurse Edward is performing discharge teaching for a newly
diagnosed diabetic patient scheduled for a fasting blood glucose test. A. Acute MI
The nurse explains to the patient that hyperglycemia is defined as a
blood glucose level above: B. CVA

A. 100 mg/dl C. Dissecting abdominal aorta

B. 120 mg/dl D. Dissecting thoracic aneurysm

C. 130 mg/dl 22. Nurse Alexandra is establishing a plan of care for a client newly
admitted with SIADH. The priority diagnosis for this client would be
D. 150 mg/dl which of the following?

17. Mang Edison is on bed rest has developed an ulcer that is full A. Fluid volume deficit
thickness and is penetrating the subcutaneous tissue. The nurse
documents that this ulcer is in which of the following stages? B. Anxiety related to disease process

A. Stage 1 C. Fluid volume excess

B. Stage 2 D. Risk for injury

C. Stage 3 23. Nursing management of the client with a UTI should include:

D. Stage 4 A. Taking medication until feeling better

18. A 24 year old male patient comes to the clinic after contracting B. Restricting fluids
genital herpes. Which of the following intervention would be most
appropriate? C. Decreasing caffeine drinks and alcohol

A. Encourage him to maintain bed rest for several days D. Douching daily
24. Felicia Gomez is 1 day postoperative from coronary artery bypass 5. B. positive homan’s sign. Pain in the calf while
surgery. The nurse understands that a postoperative patient who’s pulling up on the toes is abnormal and indicates a
maintained on bed rest is at high risk for developing: positive test. If the client feels nothing or just feels like
the calf muscle is stretching, it is considered negative.
A. angina A tourniquet test is used to measure for varicose veins.

B. arterial bleeding 6. B. Alzheimer’s disease. Alzheimer’s disease is the


most common cause of dementia in the elderly
C. deep vein thrombosis (DVT) population. AIDS, brain tumors and vascular disease
are all less common causes of progressive loss of
D. dehiscence of the wound mental function in elderly patients.

25. Which of the following statement is true regarding the visual 7. A. Apply heat compress to the affected
changes associated with cataracts? area. Options B, C and D are appropriate nursing
interventions when caring for a client diagnosed with
osteomyelitis. The application of heat can increase
A. Both eyes typically cataracts at the same time
edema and pain in the affected area and spread
bacteria through vasodilation.
B. The loss of vision is experienced as a painless, gradual
blurring
8. D. Give the Lanoxin as ordered. The Lanoxin should
be held for a pulse of 60 bpm. Nurses cannot
C. The patient is suddenly blind arbitrarily give half of a dose without a physician’s
order. Unless specific parameters are given
D. The patient is typically experiences a painful, sudden concerning pulse rate, most resources identify 60 as
blurring of vision. the reference pulse.

Answers and Rationales 9. A. To avoid heavy lifting. Heavy lifting is one factor
that leads to development of a hiatal hernia. Dietary
1. C. a nosocomial infection. Nosocomial, or hospital- factors involve limiting fat intake, not restricting
acquired are infections acquired during client to soft foods. It is more prevalent in individuals
hospitalization for which the patient isn’t being who are middle-aged or older. Fair-skinned
primarily treated. Community acquired or individuals are not prone to this condition.
opportunistic infections may not be acquired during
hospitalization. An iatrogenic infection is caused by 10. D. asterixis. Flapping tremors associated with hepatic
the doctor or by medical therapy. And an opportunistic encephalophaty are asterixis. Aphasia is the inability
infection affects a compromised host. to speak. Ascites is an accumulation of fluid in the
peritoneal cavity. Astacia is the inability to stand or sit
2. C. Assisting in ambulation to the still.
bathroom. Cerebral tissue hypoxia is commonly
associated with dizziness. The greatest potential risk 11. D. counteracts the effects of insulin. The 50%
to the client with dizziness is injury, especially with dextrose is given to counteract the effects of insulin.
changes in position. Planning for periods of rest and Insulin drives the potassium into the cell, thereby
conserving energy are important with someone with lowering the serum potassium levels. The dextrose
anemia because of his or her fatigue level but most doesn’t directly cause potassium excretion or any
important is safety. movement of potassium.

3. C. Trachea deviating to the right. A mediastinal shift 12. B. hepatomegaly. Although option D is correct, it is
is indicative of a tension pneumothorax along with the not a strong indicator of cirrhosis. Pruritus can occur
other symptoms in the question. Since the individual for many reasons. Options A and C are incorrect, fluid
was involved in a MVA, assessment would be accumulations is usually in the form of ascites in the
targeted at acute traumatic injuries to the lungs, heart abdomen. Hepatomegaly is an enlarged liver, which is
or chest wall rather than other conditions indicated in correct. The spleen may also be enlarged.
the other answers. Option A is common with
pneumonia; values in option B are not alarming; and 13. C. severe eye and face pain. Narrow-angle glaucoma
option D is typical of someone with COPD. develops abruptly and manifests with acute face and
eye pain and is a medial emergency. Halo vision, dull
4. A. away from the body. When opening an envelop- eye pain and impaired night vision are symptoms
wrapped sterile package, reaching across the package associated with open-angle glaucoma.
and using the first motion to open the top cover away
from the body eliminates the need to later reach across 14. A. hypoclacemia. Chvostek’s sign is a spasm of the
the steri9le field while opening the package. To facial muscles elicited by tapping the facial nerve and
remove equipment from the package, opening the first is associated with hypocalcemia. Clinical signs of
portion of the package toward, to the left, or to the hypokalemia are muscle weakness, leg cramps,
right of the body would require reaching across a fatigue, nausea and vomiting. Muscle cramps,
sterile field. anorexia, nausea and vomiting are clinical signs of
hyponatremia. Clinical manifestations associated with
hypophosphatemia include muscle pain, confusion, 25. B. The loss of vision is experienced as a painless,
seizures and coma. gradual blurring. Typically, a patient with cataracts
experiences painless, gradual loss of vision. Although
15. B. BUN/Crea. The BUN is primarily used as indicator both eyes may develop at different rates.
of kidney function because most renal diseases
interfere with its excretion and cause blood vessels to
rise. Creatinine is produced in relatively constant
amounts, according to the amount of muscle mass and
is excreted entirely by the kidneys making it a good
indicator of renal function.

16. B. 120 mg/dl. Hyperglycemia is defined as a blood


glucose level greater than 120 mg/dl. Blood glucose
levels of 120 mg/dl, 130 mg/dl and 150 mg/dl are
considered hyperglycemic. A blood glucose of 100
mg/dl is normal.

17. C. Stage 3. A stage 3 ulcer is full thickness involving


the subcutaneous tissue. A stage 1 ulcer has a defined
area of persistent redness in lightly pigmented skin. A
stage 2 ulcer involves partial thickness skin loss. Stage
4 ulcers extend through the skin and exhibit tissue
necrosis and muscle or bone involvement.

18. C. Instruct him to avoid sexual contact during acute


phases of illness. Herpes is a virus and is spread
through direct contact. An antifungal would not be
useful; bed rest and temperature measurement are
usually not necessary.

19. C. Patency of airway. A burn face, neck or chest may


cause airway closure because of the edema that occurs
within hours. Remember the ABC’s: airway,
breathing and circulation. Airway always comes first,
even before pain. The nurse will also assess options B
and D, but these are not the highest priority
assessments.

20. A. dizziness. Central tissue hypoxia is commonly


associated with dizziness. Recognition of cerebral
hypoxia is critical since the body will attempt to shunt
oxygenated blood to vital organs.

21. D. Dissecting thoracic aneurysm. A dissecting


thoracic aneurysm may cause loss of radical pulses
and severe chest and back pain. An MI typically
doesn’t cause loss of radial pulses or severe back pain.
CVA and dissecting abdominal aneurysm are
incorrect responses.

22. C. Fluid volume excess. SIADH results in fluid


retention and hyponatremia. Correction is aimed at
restoring fluid and electrolyte balance. Anxiety and
risk for injury should be addressed following fluid
volume excess.

23. C. Decreasing caffeine drinks and alcohol. Caffeine


and alcohol can increase bladder spasms and mucosal
irritation, thus increase the signs and symptoms of
UTI. All antibiotics should be taken completely to
prevent resistant strains of organisms.

24. C. deep vein thrombosis (DVT). DVT, is the most


probable complication for postoperative patients on
bed rest. Options A, B and D aren’t likely
complications of the post operative period.

You might also like